Brilliant Integration Contest - Season 2 (Part-1)

Hi Brilliant! Just like what Anastasiya Romanova conducted last year, this year I would also like to conduct an integration contest.

The aims of the Integration contest are to improve skills in the computation of integrals, to learn from each other as much as possible, and of course to have fun. Anyone here may participate in this contest.

The rules are as follows:

  • I will start by posting the first problem. If there is a user solves it, then they must post a new one.

  • You may only post a solution of the problem below the thread of problem and post your proposed problem in a new thread. Put them separately.

  • Only make substantial comment that will contribute to the discussion.

  • Make sure you know how to solve your own problem before posting it in case there is no one can answer it within 48 hours, then you must post the solution and you have a right to post another problem.

  • If the one who solves the last problem does not post his/her own problem after solving it within a day, then the one who has a right to post a problem is the last solver before him/her.

  • The scope of questions is only computation of integrals either definite or indefinite integrals.

  • You are NOT allowed to post a multiple integrals problem.

  • It is NOT compulsory to post original problems. But make sure it has not been posted on brilliant.

  • Do not copy questions from last year's contest. If anyone found to do so he/she will be banned from taking further part in this contest

  • You are also NOT allowed to post a solution using a contour integration or residue method.

The final answer can ONLY contain the following special functions: gamma function, beta function, Riemann zeta function, Dirichlet eta function, dilogarithm, digamma function, trigonometric integral, Wallis' integral, Bessel function, contour integration and Ramanujan's Master Theorem (including Mellin Transform).

Please post your solution and your proposed problem in a single new thread.

Format your post is as follows:

1
2
3
4
5
6
7
**SOLUTION OF PROBLEM xxx (number of problem) :**

**[Post your solution here]**

**PROBLEM xxx (number of problem) :**

**[Post your problem here]**

The comments will be easiest to follow if you sort by "Newest":

View Part 2

#Calculus #Integration #IntegrationTechniques #Integrals #Interesting

Note by Aditya Kumar
5 years, 6 months ago

No vote yet
1 vote

  Easy Math Editor

This discussion board is a place to discuss our Daily Challenges and the math and science related to those challenges. Explanations are more than just a solution — they should explain the steps and thinking strategies that you used to obtain the solution. Comments should further the discussion of math and science.

When posting on Brilliant:

  • Use the emojis to react to an explanation, whether you're congratulating a job well done , or just really confused .
  • Ask specific questions about the challenge or the steps in somebody's explanation. Well-posed questions can add a lot to the discussion, but posting "I don't understand!" doesn't help anyone.
  • Try to contribute something new to the discussion, whether it is an extension, generalization or other idea related to the challenge.
  • Stay on topic — we're all here to learn more about math and science, not to hear about your favorite get-rich-quick scheme or current world events.

MarkdownAppears as
*italics* or _italics_ italics
**bold** or __bold__ bold

- bulleted
- list

  • bulleted
  • list

1. numbered
2. list

  1. numbered
  2. list
Note: you must add a full line of space before and after lists for them to show up correctly
paragraph 1

paragraph 2

paragraph 1

paragraph 2

[example link](https://brilliant.org)example link
> This is a quote
This is a quote
    # I indented these lines
    # 4 spaces, and now they show
    # up as a code block.

    print "hello world"
# I indented these lines
# 4 spaces, and now they show
# up as a code block.

print "hello world"
MathAppears as
Remember to wrap math in \( ... \) or \[ ... \] to ensure proper formatting.
2 \times 3 2×3 2 \times 3
2^{34} 234 2^{34}
a_{i-1} ai1 a_{i-1}
\frac{2}{3} 23 \frac{2}{3}
\sqrt{2} 2 \sqrt{2}
\sum_{i=1}^3 i=13 \sum_{i=1}^3
\sin \theta sinθ \sin \theta
\boxed{123} 123 \boxed{123}

Comments

Problem 1:

Evaluate: 01xln(x)1x2dx\int _{ 0 }^{ 1 }{\frac { x \ln\left( x \right) }{ \sqrt { 1-{ x }^{ 2 } } } \, dx }

This problem has been solved by Surya Prakash.

Aditya Kumar - 5 years, 6 months ago

Log in to reply

Another solution: Let (1x2)=t\sqrt{(1-x^2)}=t

x(1x2)dx=dt\frac{-x}{\sqrt{(1-x^2)}}dx=dt

So integration becomes

10ln(1t2)dt- \int_1^0 ln\sqrt{(1-t^2)}dt

1210ln(1t2)dt-\frac{1}{2} \int_1^0 ln(1-t^2)dt

1201ln(1t2)dt\frac{1}{2} \int_0^1 ln(1-t^2)dt

1201(ln(1t)+ln(1+t))dt\frac{1}{2} \int_0^1 (ln(1-t)+ln(1+t))dt

This can be easily evaluated and comes out to be ln21ln2-1

Gautam Sharma - 5 years, 6 months ago

Solution to Problem 1:\Large \text{Solution to Problem 1:}

01xln(x)1x2dx\int_{0}^{1} \dfrac{x \ln(x)}{\sqrt{1-x^2}} dx Take t=arcsin(x)    dt=dx1x2t = \arcsin(x) \iff dt = \dfrac{dx}{\sqrt{1-x^2}} and the limits changes to t=0t=0 to t=π2t=\dfrac{\pi}{2}. 0π2sin(t)ln(sin(t))dt\int_{0}^{\dfrac{\pi}{2}} \sin (t) \ln(\sin(t)) dt

Consider,

B(x,y)=01tx1(1t)y1dtB(x,y) = \int_{0}^{1} t^{x-1} (1-t)^{y-1} dt

Taking the transformation t=sin2(z)    dt=2sin(z)cos(z)dzt=\sin^2 (z) \iff dt = 2\sin(z) \cos (z)dz, the integral transforms into

B(x,y)=20π/2sin2x1(z)cos2y1(z)dzB(x,y) = 2 \int_{0}^{\pi /2} \sin ^{2x-1} (z) \cos^ {2y-1} (z) dz

But B(x,y)=Γ(x)Γ(y)Γ(x+y)B(x,y) = \dfrac{\Gamma (x) \Gamma(y) }{\Gamma (x+y)}

So, 0π/2sin2x1(z)cos2y1(z)dz=12Γ(x)Γ(y)Γ(x+y)\int_{0}^{\pi /2} \sin ^{2x-1} (z) \cos^ {2y-1} (z) dz = \dfrac{1}{2} \dfrac{\Gamma (x) \Gamma(y) }{\Gamma (x+y)}

Differentiate it w.r.t xx

20π/2sin2x1(z)cos2y1(z)ln(sin(z))dz=12xΓ(x)Γ(y)Γ(x+y)0π/2sin2x1(z)cos2y1(z)ln(sin(z))dz=14Γ(x)Γ(y)Γ(x+y)(ψ(x)ψ(x+y))2 \int_{0}^{\pi /2} \sin ^{2x-1} (z) \cos^ {2y-1} (z) \ln(\sin(z)) dz = \dfrac{1}{2} \dfrac{\partial}{\partial x} \dfrac{\Gamma (x) \Gamma(y) }{\Gamma (x+y)} \\ \int_{0}^{\pi /2} \sin ^{2x-1} (z) \cos^ {2y-1} (z) \ln(\sin(z)) dz = \dfrac{1}{4} \dfrac{\Gamma (x) \Gamma(y) }{\Gamma (x+y)} (\psi (x) - \psi(x+y))

Take x=1x= 1 and y=12y= \dfrac{1}{2}.

0π/2sin(z)ln(sin(z))dz=14Γ(1)Γ(1/2)Γ(3/2)(ψ(1)ψ(3/2))=ln(2)1\int_{0}^{\pi / 2} \sin(z) \ln (\sin (z)) dz = \dfrac{1}{4} \dfrac{\Gamma (1) \Gamma(1/2) }{\Gamma (3/2)} (\psi (1) - \psi(3/2)) = \ln (2) -1

Surya Prakash - 5 years, 6 months ago

Problem 2:

Evaluate:

0xcos(x3)exp(x3)dx\Large \int_{0} ^{\infty} \dfrac{x \cos (x^3)}{\exp (x^3)} \, dx

This problem has been solved by Tanishq Varshney.

Surya Prakash - 5 years, 6 months ago

Log in to reply

Solution to Problem 2\large{Solution~to~Problem~ 2}

Firstly I would add a proof of my solution

0eaxxn1dx=Γ(n)an\large{\displaystyle \int _{0}^{\infty} e^{-ax}x^{n-1} dx=\frac{\Gamma (n)}{a^{n}}} where Γ(n)\Gamma (n) is Gamma function

Replace aa+iba \rightarrow a+ib where i=1i =\sqrt{-1}

0eaxeibxxn1dx=Γ(n)(a+ib)n\large{\displaystyle \int _{0}^{\infty} e^{-ax}e^{-ibx}x^{n-1} dx=\frac{\Gamma (n)}{(a+ib)^{n}}}

Put a=rcosy and b=rsinya=r \cos y ~ and ~ b=r \sin y.

So that r2=a2+b2r^2=a^2+b^2 and y=arctan(ba)y=\arctan \left(\frac{b}{a}\right)

Using de moviers theorem

0eax(cosbxisinbx)xn1dx=Γ(n)rn(cosny+sinny)1\large{\displaystyle \int _{0}^{\infty} e^{-ax}(\cos bx-i \sin bx)x^{n-1} dx=\frac{\Gamma (n)}{r^{n}}(\cos ny+\sin ny)^{-1}}

Comparing the real and imaginary parts we finally get

0xn1eax(cosbx)dx=Γ(n)rn(cosny)\large{\displaystyle \int _{0}^{\infty}x^{n-1} e^{-ax}(\cos bx) dx=\frac{\Gamma (n)}{r^{n}}(\cos ny)}

now in the given integral put x3=tx^{3}=t

The integral now becomes

130t13etcos(t)dt\large{\frac{1}{3} \displaystyle \int^{\infty}_{0} t^{-\frac{1}{3}}e^{-t} \cos (t) dt}

here n=23n=\frac{2}{3};a=1a=1 and b=1b=1

Thus we get 13Γ(23)cos(π6)213\large{\frac{1}{3}\frac{\Gamma \left(\frac{2}{3}\right) \cos \left(\frac{\pi}{6}\right)}{2^{\frac{1}{3}}}}

3Γ(23)3.243\large{\boxed{\frac{\sqrt{3} \Gamma \left(\frac{2}{3}\right)}{3.2^{\frac{4}{3}}}}}

Tanishq Varshney - 5 years, 6 months ago

Problem 23:

Evaluate:01log(1+x)log(1x3)dx\int _{ 0 }^{ 1 }{ \log\left( 1+x \right) \log\left( 1-{ x }^{ 3 } \right) dx }

Due to time constraint, Aditya Kumar decided to post a solution himself.

Aditya Kumar - 5 years, 6 months ago

Log in to reply

Solution to Problem 23:

Substitute: x1x1+xx\longrightarrow \frac { 1-x }{ 1+x }

I=01ln(1+x)ln(1x3)dx=201ln(2x(3+x2)(1+x)3)ln(21+x)dx(1+x)2I=\int_0^1 \ln(1+x)\ln(1-x^3)dx=2\int_0^1 \ln\left(\frac{2 x (3+x^2)}{(1+x)^3}\right)\ln\left(\frac{2}{1+x}\right)\frac{dx}{(1+x)^2}

On separating the integrands,

ln(2x(3+x2)(1+x)3)ln(21+x)=ln22ln2lnx+ln2ln(3+x2)4ln2ln(1+x)lnxln(1+x)+3ln2(1+x)ln(1+x)ln(3+x2)\ln\left(\frac{2 x (3+x^2)}{(1+x)^3}\right)\ln\left(\frac{2}{1+x}\right) \\\small =\ln^22-\ln2\ln x+\ln2\ln(3+x^2)-4\ln2\ln(1+x)-\ln x\ln(1+x)+3\ln^2(1+x)-\ln(1+x)\ln(3+x^2)

1st integral:

\int_0^1 \frac{dx}{(1+x)^2}=-\frac1{1+x}\Bigg{|}_0^1=\frac12

2nd integral:

01lnx(1+x)2dx=n1(1)nn=ln2\int_0^1 \frac{\ln x}{(1+x)^2}dx=\sum_{n\geq1} \frac{(-1)^n}{n}=-\ln2

3rd integral:

\int_0^1 \frac{\ln(1+x)}{(1+x)^2}dx=-\frac{\ln(1+x)}{1+x}\Bigg{|}_0^1+\int_0^1\frac{dx}{(1+x)^2}dx=\frac12-\frac{\ln2}{2}

4th integral:

\int_0^1 \frac{\ln(3+x^2)}{(1+x)^2}dx=-\frac{\ln(3+x^2)}{1+x}\Bigg{|}_0^1+\int_0^1 \frac{2x}{(1+x)(3+x^2)}dx \\=-\ln2+\ln3+2\operatorname{Re} \int_0^1 \frac{dx}{(1+x)(x+i\sqrt{3})}=-\ln2+\frac34\ln3+\frac{\pi}{4\sqrt{3}}

5th integral:

\int_0^1 \frac{\ln x \ln(1+x)}{(1+x)^2}dx=-\frac{\ln x\ln(1+x)}{1+x}\Bigg{|}_0^1+\int_0^1\frac1{1+x}\left(\frac{\ln(1+x)}{x}+\frac{\ln x}{1+x}\right)dx \\=-\ln2+\int_0^1 \frac{\ln(1+x)}{x}dx-\int_0^1\frac{\ln(1+x)}{1+x}dx=\frac{\pi^2}{12}-\frac12\ln^22-\ln2

6th integral:

\int_0^1 \frac{\ln^2(1+x)}{(1+x)^2}dx=-\frac{\ln^2(1+x)}{1+x}\Bigg{|}_0^1+\int_0^1 \frac{2\ln(1+x)}{(1+x)^2}dx \\=1-\ln2-\frac12\ln^22

7th integral:

\int_0^1 \frac{\ln(1+x)\ln(3+x^2)}{(1+x)^2}dx=-\frac{\ln(1+x)\ln(3+x^2)}{1+x}\Bigg{|}_0^1+\int_0^1 \frac1{1+x}\left(\frac{2x\ln(1+x)}{3+x^2}+\frac{\ln(3+x^2)}{1+x}\right)dx \\=-\ln^22-\ln2+\frac34\ln3+\frac{\pi}{4\sqrt{3}}+2J

where,

J=01xln(1+x)(1+x)(3+x2)dx=Re01ln(1+x)(1+x)(x+i3)dx=18ln22Re1i3101ln(1+x)x+i3dxJ=\int_0^1\frac{x\ln(1+x)}{(1+x)(3+x^2)}dx=\operatorname{Re} \int_0^1 \frac{\ln(1+x)}{(1+x)(x+i\sqrt{3})}dx \\=-\frac18\ln^22-\operatorname{Re} \,\frac1{i\sqrt{3}-1}\int_0^1 \frac{\ln(1+x)}{x+i\sqrt{3}}dx

Now on using the formula:

01ln(1+x)x+adx=ln2lna+1a1+Li2(21a)Li2(11a)\int_0^1 \frac{\ln(1+x)}{x+a}dx=\ln2 \ln\frac{a+1}{a-1}+\operatorname{Li}_2\left(\frac2{1-a}\right)-\operatorname{Li}_2\left(\frac1{1-a}\right)

Putting a=i3a=i\sqrt{3} then gives, after taking the real part, and assuming the principle value of the logarithm,

J=18ln22+π312ln2+14ReLi2(eiπ/3)34ImLi2(eiπ/3)14ReLi2(12eiπ/3)+34ImLi2(12eiπ/3)\small J=-\frac18\ln^22+\frac{\pi\sqrt{3}}{12}\ln2+\frac14\operatorname{Re}\text{Li}_2(e^{i\pi/3})-\frac{\sqrt{3}}{4}\operatorname{Im}\text{Li}_2(e^{i\pi/3})-\frac14\operatorname{Re}\text{Li}_2(\frac12 e^{i\pi/3})+\frac{\sqrt{3}}{4}\operatorname{Im}\text{Li}_2(\frac12 e^{i\pi/3})

Using:

ReLi2(eiπ/3)=π236\displaystyle \,\, \operatorname{Re}\text{Li}_2(e^{i\pi/3})=\frac{\pi^2}{36},

ImLi2(eiπ/3)=123ψ1(13)π233\displaystyle \,\, \operatorname{Im}\text{Li}_2(e^{i\pi/3})=\frac1{2\sqrt{3}}\psi_1\left(\frac13\right)-\frac{\pi^2}{3\sqrt{3}},

Li2(12eiπ/3)=Li2(i3)12ln2(34i34)\displaystyle \,\,\text{Li}_2(\frac12 e^{i\pi/3})=-\text{Li}_2\left(-\frac{i}{\sqrt{3}}\right)-\frac12\ln^2\left(\frac34-i\frac{\sqrt{3}}{4}\right),

and

ReLi2(i3)=n=1(1)n(2n)23n=14Li2(13)\displaystyle \,\, \operatorname{Re}\text{Li}_2\left(-\frac{i}{\sqrt{3}}\right)=\sum_{n=1}^{\infty} \frac{(-1)^n}{(2n)^2\, 3^n}=\frac14\text{Li}_2\left(-\frac13\right)

we have ReLi2(12eiπ/3)=π27218ln2(34)14Li2(13)\operatorname{Re}\text{Li}_2(\frac12 e^{i\pi/3})=\frac{\pi^2}{72}-\frac18\ln^2\left(\frac34\right)-\frac14\text{Li}_2\left(-\frac13\right)

and ImLi2(12eiπ/3)=π12ln(34)ImLi2(i3).\operatorname{Im}\text{Li}_2(\frac12 e^{i\pi/3})=\frac{\pi}{12}\ln\left(\frac34\right)-\operatorname{Im}\text{Li}_2\left(-\frac{i}{\sqrt{3}}\right).

Finally, putting everything together gives: I=ln2218ln23+2ln2ln332ln36ln2+6π43(2+ln3)37π272+12ψ1(13)14Li2(13)+3Li2(i3)I=\ln^22 - \frac18\ln^23 + 2\ln2\ln3 - \frac32\ln3 - 6\ln2 + 6-\frac{\pi}{4\sqrt{3}}(2 + \ln3) - \frac{37\pi^2}{72} + \frac12\psi_1\left(\frac13 \right ) - \frac14\text{Li}_2\left( -\frac13 \right ) + \sqrt{3}\Im\text{Li}_2\left( -\frac{i}{\sqrt{3}} \right )

......

This was not original. I found it very awesome, so I thought to share it here.

Aditya Kumar - 5 years, 6 months ago

Log in to reply

I don't know what's wrong with the rendering of the Latex, since it sometimes render and sometimes it doesn't. So I'm going to post in pictures the one's that didn't render. I think it's better to move this discussion to another note.

1st Integral:

3rd Integral:

4th Integral:

5th Integral:

6th Integral:

7th Integral:

Julian Poon - 5 years, 6 months ago

Whoops, I accidentally deleted Tanishq's comment:

Problem 5:

0π/2tan1(1729cos2x)dx \int_0^{\pi /2} \tan^{-1} (1729\cos^2 x) \, dx

This problem has been solved by Sudeep Salgia.

Pi Han Goh - 5 years, 6 months ago

Log in to reply

I'll post the outline of the solution as it is a bit cumbersome. Let
I(a)=0π2arctan(acos2x)dxI(a)=cos2x1+a2cos4xdx=sec2xsec4x+a2dx \displaystyle I(a) = \int_0^{\frac{\pi}{2}} \arctan (a \cos^2 x) dx \Rightarrow I'(a) = \frac{\cos^2 x}{1 + a^2 \cos^4 x } dx = \frac{\sec^2 x}{\sec^4 x + a^2 } dx Substitute tanx=t \tan x = t , to get I(a)=01t4+2t2+(a2+1)dt \displaystyle I'(a) = \int_0^{\infty} \frac{1}{t^4 + 2t^2 + (a^2 + 1)} dt . Put a2+1=k2a^2 + 1 = k^2 and with some simple rearrangement, we get,
2kI(a)=01+kt2(tkt)2+2(k+1)01kt2(t+kt)2+2(k+1)\displaystyle 2k I'(a) = \int_0^{\infty} \frac{ 1+ \frac{k}{t^2}}{\left( t - \frac{k}{t} \right)^2 + 2(k+1)} - \int_0^{\infty} \frac{ 1- \frac{k}{t^2}}{\left( t + \frac{k}{t} \right)^2 + 2(-k+1)} Substitute tkt=z \displaystyle t - \frac{k}{t} = z and t+kt=y \displaystyle t + \frac{k}{t} = y in the respective integrals and evaluate them easily (They come out to be arctan \arctan of something ). After all this we can write,
I(a)=π221a2+11a2+1+1da\displaystyle I(a) = \frac{\pi}{2\sqrt{2}} \int \frac{1}{\sqrt{a^2 + 1}} \frac{1}{\sqrt{\sqrt{a^2 + 1} +1}} da
This can be rearranged to be written as
I(a)=π2211+a2+112×12a2+112×12×aa2+1da \displaystyle I(a) = \frac{\pi}{2\sqrt{2}} \int \frac{1}{1+ \frac{\sqrt{a^2 + 1} - 1 }{2}} \times \frac{1}{2\sqrt{ \frac{\sqrt{a^2 + 1} - 1 }{2} }} \times \frac{1}{2} \times \frac{a}{\sqrt{a^2 +1}} da

This evaluates to π2arctan(a2+112) \displaystyle \frac{\pi}{2} \arctan \left( \sqrt{ \frac{\sqrt{a^2+1} - 1}{2}} \right)

The constant of integration is 0 0 from I(0)=0 I(0) = 0 . Put a=1729 a =1729 to get the answer.

Sudeep Salgia - 5 years, 6 months ago

Problem 24:

Prove that

0π/21sin8x+cos8xdx=1022π. \large \int_0^{\pi /2} \dfrac1{\sin^8 x + \cos^8 x } \, dx = \dfrac{\sqrt{10-\sqrt2}}{2} \pi .

This problem has been solved by Tanishq Varshney.

Pi Han Goh - 5 years, 6 months ago

Log in to reply

Solution to Problem 24\text{Solution to Problem 24}

0xm11+xdx=πsinmπ0<m<1\large{\displaystyle \int ^{\infty}_{0} \frac{x^{m-1}}{1+x}dx=\frac{\pi}{\sin m \pi} \quad \quad 0<m<1}

Refer to problem 18 for this step.

Now in the integral tanxt\tan x \rightarrow t

0(1+t2)31+t8dt\large{\displaystyle \int ^{\infty}_{0} \frac{(1+t^2)^3}{1+t^{8}}dt}

t8z\large{t^8 \rightarrow z}

18(0z7811+z+3z5811+z+3z3811+z+z1811+zdz)\large{\frac{1}{8} \left(\displaystyle \int ^{\infty}_{0} \frac{z^{\frac{7}{8}-1}}{1+z}+ \frac{3z^{\frac{5}{8}-1}}{1+z}+ \frac{3z^{\frac{3}{8}-1}}{1+z}+ \frac{z^{\frac{1}{8}-1}}{1+z} dz \right)}

π4(1sin(π8)+3cos(π8))\large{\rightarrow \frac{\pi}{4} \left(\frac{1}{\sin \left(\frac{\pi}{8} \right)}+\frac{3}{\cos \left(\frac{\pi}{8} \right)} \right)}

=1022π\large{=\frac{\sqrt{10-\sqrt{2}}}{2} \pi}

Tanishq Varshney - 5 years, 6 months ago

Problem 3\text{Problem 3}

Evaluate:

0π2tan35xdx\large {\displaystyle \int^{\frac{\pi}{2}}_{0} \tan^{\frac{3}{5}} x \, dx}

This problem has been solved by Aditya Kumar.

Tanishq Varshney - 5 years, 6 months ago

Log in to reply

Solution to Problem 3: 0π2(tanx)35dx=0π2(sinx)35(cosx)35dx=B(45,15)2...........{B(x,y)isbetafunction}=Γ(45)Γ(15)2Γ(1)=π2sin(π5)...............{byeulersreflectionformula}=2π1025\int _{ 0 }^{ \frac { \pi }{ 2 } }{ { \left( tanx \right) }^{ \frac { 3 }{ 5 } } } dx=\int _{ 0 }^{ \frac { \pi }{ 2 } }{ { \left( sinx \right) }^{ \frac { 3 }{ 5 } }{ \left( cosx \right) }^{ \frac { -3 }{ 5 } } } dx\\ \quad \quad \quad \quad \quad \quad \quad \quad \quad \quad \quad =\quad \frac { B\left( \frac { 4 }{ 5 } ,\frac { 1 }{ 5 } \right) }{ 2 } ...........\left\{ B(x,y)\quad is\quad beta\quad function \right\} \\ \quad \quad \quad \quad \quad \quad \quad \quad \quad \quad \quad =\frac { \Gamma \left( \frac { 4 }{ 5 } \right) \Gamma \left( \frac { 1 }{ 5 } \right) }{ 2\Gamma \left( 1 \right) } \\ \quad \quad \quad \quad \quad \quad \quad \quad \quad \quad \quad =\frac { \pi }{ 2sin\left( \frac { \pi }{ 5 } \right) } \quad ...............\{ by\quad euler's\quad reflection\quad formula\} \\ \quad \quad \quad \quad \quad \quad \quad \quad \quad \quad \quad =\frac { 2\pi }{ \sqrt { 10-2\sqrt { 5 } } }

Aditya Kumar - 5 years, 6 months ago

Problem 4:

Let P(x)P(x) be a polynomial such that P(x+1)P(x)=x2+x+1x2x+1\frac { P\left( x+1 \right) }{ P\left( x \right) } =\frac { { x }^{ 2 }+x+1 }{ { x }^{ 2 }-x+1 } and P(2)=3P(2)=3, evaluate01(tan1(P(x)).tan1(x1x))dx.\int _{ 0 }^{ 1 }{ \left( { \tan }^{ -1 }\left( P(x) \right) .{ \tan }^{ -1 }\left( \sqrt { \frac { x }{ 1-x } } \right) \right) \, dx }.

This problem has been solved by Tanishq Varshney.

Aditya Kumar - 5 years, 6 months ago

Log in to reply

Solution to Problem 4\text{Solution to Problem 4}

Clearly P(x)=x2x+1P(x)=x^2-x+1 and P(1x)=P(x)P(1-x)=P(x)

Also tan1(x)=cot1(1x)\tan^{-1} (x)=\cot^{-1} \left(\frac{1}{x}\right)

tan1(x)+cot1(x)=π2\tan^{-1}(x)+\cot^{-1}(x)=\frac{\pi}{2}

I=01(tan1(P(x))cot1(x1x))dx\large{I=\displaystyle \int^{1}_{0} \left( \tan^{-1}(P(x)) \cot^{-1} \left( \sqrt{\frac{x}{1-x}}\right) \right) dx}

2I=π201tan1(x2x+1)dx\large{2I=\frac{\pi}{2} \displaystyle \int^{1}_{0}\tan^{-1} (x^2-x+1) dx}

2I=π2(π201tan1(11+x(x1)))\large{2I=\frac{\pi}{2}\left(\frac{\pi}{2} - \displaystyle \int^{1}_{0} \tan^{-1}\left(\frac{1}{1+x(x-1)}\right)\right)}

2I=π2(π201tan1(x(x1)1+x(x1))dx)\large{2I=\frac{\pi}{2}\left(\frac{\pi}{2} - \displaystyle \int^{1}_{0} \tan^{-1}\left(\frac{x-(x-1)}{1+x(x-1)}\right) dx \right)}

2I=π2(π2(01tan1(x)dx01tan1(x1)dx))\large{2I=\frac{\pi}{2}\left(\frac{\pi}{2} -( \displaystyle \int^{1}_{0} \tan^{-1} (x) dx- \displaystyle \int^{1}_{0} \tan^{-1} (x-1) dx)\right)}

2I=π2(π2(π22ln(2)))\large{2I=\frac{\pi}{2}\left(\frac{\pi}{2} -\left( \frac{\pi}{2}-2 \ln (2)\right)\right)}

I=π2ln(2)\large{I=\frac{\pi}{2} \ln(2)}

Tanishq Varshney - 5 years, 6 months ago

Problem 9:

Evaluate the indefinite integral: x2+20(xsinx+5cosx)2dx.\large{\displaystyle \int \frac{x^2+20}{(x \sin x+5 \cos x)^2} \, dx}.

This problem has been solved by Surya Prakesh.

Tanishq Varshney - 5 years, 6 months ago

Log in to reply

x2+20=(xsin(x)+5cos(x))2+(5sin(x)xcos(x))25=(xsin(x)+5cos(x))2+(5sin(x)xcos(x))25cos2(x)5sin2(x)xcos(x)sin(x)+xcos(x)sin(x)=(xsin(x)+5cos(x))(5cos(x)+xsin(x)cos(x))+(5sin(x)xcos(x))(5sin(x)xcos(x)sin(x))=(xsin(x)+5cos(x))(5cos(x)+xsin(x)cos(x))(5sin(x)xcos(x))(xcos(x)+sin(x)5sin(x))=(xsin(x)+5cos(x))d(5sin(x)xcos(x))dx(5sin(x)xcos(x))d(xsin(x)+5cos(x))dx\begin{aligned} x^2 + 20 &= (x \sin(x) + 5 \cos(x))^2 + (5 \sin (x) - x \cos(x))^2 - 5 \\ &=(x \sin(x) + 5 \cos(x))^2 + (5 \sin (x) - x \cos(x))^2 - 5 \cos^2(x) - 5\sin^2 (x) -x \cos(x)\sin(x) + x\cos(x) \sin(x) \\ &= (x \sin(x) + 5 \cos(x)) ( 5 \cos(x) + x \sin(x) - \cos(x)) + (5 \sin(x) - x \cos(x))(5 \sin(x) - x \cos(x) - \sin(x)) \\ &= (x \sin(x) + 5 \cos(x)) ( 5 \cos(x) + x \sin(x) - \cos(x)) - (5 \sin(x) - x \cos(x))( x \cos(x) + \sin(x) - 5 \sin (x)) \\ &= (x \sin(x) + 5 \cos(x)) \dfrac{d(5 \sin(x) - x \cos(x))}{dx} - (5 \sin(x) - x \cos(x)) \dfrac{d(x \sin(x) + 5 \cos(x))}{dx} \end{aligned}

Let u=xsin(x)+5cos(x)u=x \sin(x) + 5 \cos(x) and v=5sin(x)xcos(x)v=5 \sin(x) - x \cos(x). So, x2+20=udvdxvdudx=u2d(vu)dxx^2 + 20 = u\dfrac{dv}{dx} - v\dfrac{du}{dx} = u^2 \dfrac{d\left(\dfrac{v}{u}\right)}{dx}.

So, x2+20(xsin(x)+5cos(x))2dx=u2d(vu)dxu2dx=vu=5sin(x)xcos(x)xsin(x)+5cos(x)\int \dfrac{x^2 + 20}{(x \sin(x) + 5 \cos(x))^2} dx = \int \dfrac{ u^2 \dfrac{d\left(\dfrac{v}{u}\right)}{dx}}{u^2} dx= \dfrac{v}{u} = \dfrac{5 \sin(x) - x \cos(x)}{x \sin(x) + 5 \cos(x)}

Surya Prakash - 5 years, 6 months ago

Problem 13: (Originally posted by Tanishq)

Evaluate 01xln(1+x1x)dx. \int_0^1 x \ln \left( \dfrac{1+x}{1-x} \right) \, dx.

This problem has been solved by Surya Prakesh.

Pi Han Goh - 5 years, 6 months ago

Log in to reply

By using integration by parts,

xln(1+x1x)dx=x22ln(1+x1x)+x12ln(1+x)+12ln(1x)\int x \ln \left(\dfrac{1+x}{1-x} \right) dx = \dfrac{x^2}{2} \ln \left(\dfrac{1+x}{1-x} \right) + x - \dfrac{1}{2} \ln \left(1+x \right) + \dfrac{1}{2} \ln \left(1-x \right)

So,

01xln(1+x1x)dx=lima001axln(1+x1x)dx=lima0(1a)22ln(2aa)+1a12ln(2a)+12lna=1\begin{aligned} \int_{0}^{1} x \ln \left(\dfrac{1+x}{1-x} \right) dx &= \lim_{a \rightarrow 0} \int_{0}^{1-a} x \ln \left(\dfrac{1+x}{1-x} \right) dx \\ &= \lim_{a \rightarrow 0} \dfrac{(1-a)^2}{2} \ln \left(\dfrac{2-a}{a} \right) + 1 - a - \dfrac{1}{2} \ln \left(2-a \right) + \dfrac{1}{2} \ln a \\ &= 1\end{aligned}

Surya Prakash - 5 years, 6 months ago

Problem 6:
Show that for positive reals, a,b a , b where a<b a<b, abarccos(x(a+b)xab)dx=π4(ba)2a+b \int_a^b \arccos \left( \frac{x}{\sqrt{(a+b)x - ab}} \right) \, dx = \frac{\pi}{4} \frac{ (b-a)^2}{a+b}

Statutory warning: The substitutions need not be very intuitive.

This problem has been solved Aditya Kumar.

Sudeep Salgia - 5 years, 6 months ago

Log in to reply

Solution to Problem 6:

I=abarccos(x(a+b)xab)dxI=abarccos(xab((a+b)ab)x1)dx((a+b)ab)x1=tI=abba(2ta+barccos((t2+1)abt))dtI=abba(2ta+barccos((t+1t)ab))dtI=\int _{ a }^{ b }{ \text{arccos}\left( \frac { x }{ \sqrt { \left( a+b \right) x-ab } } \right) dx } \\ I=\int _{ a }^{ b }{ \text{arccos}\left( \frac { \frac { x }{ \sqrt { ab } } }{ \sqrt { \left( \frac { \left( a+b \right) }{ ab } \right) x-1 } } \right) dx } \\ \sqrt { \left( \frac { \left( a+b \right) }{ ab } \right) x-1 } =t\\ I=\int _{ \sqrt { \frac { a }{ b } } }^{ \sqrt { \frac { b }{ a } } }{ \left( \frac { 2t }{ a+b } arccos\left( \frac { \left( { t }^{ 2 }+1 \right) \sqrt { ab } }{ t } \right) \right) dt } \\ I=\int _{ \sqrt { \frac { a }{ b } } }^{ \sqrt { \frac { b }{ a } } }{ \left( \frac { 2t }{ a+b } arccos\left( \left( { t }+\frac { 1 }{ t } \right) \sqrt { ab } \right) \right) dt }

Now using Integration By Parts,

I=abba(t2+1(baab)2(t1t)2)dtt=p+1+p2I=12(ba+ab)12(baab)(2p2ab+ba+24p2)dpI=\int _{ \sqrt { \frac { a }{ b } } }^{ \sqrt { \frac { b }{ a } } }{ \left( \frac { { t }^{ 2 }+1 }{ { \left( \sqrt { \frac { b }{ a } } -\sqrt { \frac { a }{ b } } \right) }^{ 2 }-{ \left( { t }-\frac { 1 }{ t } \right) }^{ 2 } } \right) dt } \\ t=p+\sqrt { 1+{ p }^{ 2 } } \\ I=\int _{ \frac { 1 }{ 2 } \left( -\sqrt { \frac { b }{ a } } +\sqrt { \frac { a }{ b } } \right) }^{ \frac { 1 }{ 2 } \left( \sqrt { \frac { b }{ a } } -\sqrt { \frac { a }{ b } } \right) }{ \left( \frac { { 2p }^{ 2 } }{ \sqrt { \frac { \frac { a }{ b } +\frac { b }{ a } +2 }{ 4 } -{ p }^{ 2 } } } \right) dp }

Now,

p2=(ab+ba+24)sinθ{ p }^{ 2 }=\left( \frac { \frac { a }{ b } +\frac { b }{ a } +2 }{ 4 } \right) sin\theta

This can be evaluated easily.

I=π4(ba)2(a+b)\therefore \quad I=\frac { \pi }{ 4 } \frac { { \left( b-a \right) }^{ 2 } }{ \left( a+b \right) }

Moral of the question:
To whatever extent we learn anything, we must never forget our basics.
Note that this question is solved using very basic concepts.

Any elegant modification to the solution will be accepted.

Aditya Kumar - 5 years, 6 months ago

Problem 8:

Evaluate

f(ω)=12πea2t2eiωtdt\large f(\omega)=\frac{1}{\sqrt{2\pi}}\int_{-\infty}^{\infty}e^{-a^2t^2}e^{i\omega t}dt

This problem has been solved by both Tanishq Varshney (first) and Surya Prakash (second) almost at the same time.

Abhishek Bakshi - 5 years, 6 months ago

Log in to reply

Solution to problem 8\large{\text{Solution to problem 8}}

ttt \rightarrow -t

adding this with the original equation

2f(w)=22πea2t2coswtdt\Large{2f(w)=\frac{2}{\sqrt{2 \pi}} \displaystyle \int^{\infty}_{-\infty}e^{-a^2 t^2} \cos wt dt}

as eiwt+eiwt=2coswt=2(eiwt)e^{iwt}+e^{-iwt}=2 \cos wt=2 \Re(e^{iwt})

f(w)=12π(ea2t2eiwtdt)\Large{f(w)=\frac{1}{\sqrt{2 \pi}} \Re \left(\displaystyle \int^{\infty}_{-\infty}e^{-a^2 t^2}e^{iwt} dt \right)}

Let at=zat=z

f(w)=1a2π(ez2eiwazdz)\Large{f(w)=\frac{1}{a \sqrt{2 \pi}} \Re \left(\displaystyle \int^{\infty}_{-\infty}e^{-z^2}e^{i\frac{w}{a}z} dz \right)}

f(w)=1a2π(e(ziw2a)2ew24a2dz)\Large{f(w)=\frac{1}{a \sqrt{2 \pi}} \Re \left(\displaystyle \int^{\infty}_{-\infty}e^{-(z-\frac{iw}{2a})^2}e^{-\frac{w^2}{4a^2}} dz \right)}

Using Gaussian integral

f(w)=1a2ew24a2\Large{f(w)=\frac{1}{a \sqrt{2}} e^{-\frac{w^2}{4a^2}}}

Tanishq Varshney - 5 years, 6 months ago

f(ω)=12πea2t2eiωtdtf(\omega) = \dfrac{1}{\sqrt{2 \pi}} \int_{- \infty} ^{\infty} e^{-a^2 t^2} e^{i\omega t}dt

It implies that f(ω)=[F(F)](ω)f(\omega) = \big[\mathcal{F}(F) \big](\omega), where F(t)=ea2t2F(t) = e^{-a^2 t^2}.

Now, differentiate F(t)F(t), we get F(t)=2a2tF(t)F'(t) = -2a^2 t F(t).

Applying fourier transformation on both sides gives iωf(ω)=2ia2f(ω)i \omega f(\omega) = -2i a^2 f'(\omega).

    ωf(ω)=2a2df(ω)dω    f(ω)=ceω24a2\implies \omega f(\omega) = -2a^2 \dfrac{d f(\omega)}{d \omega} \\ \implies f(\omega) = c e^{-\dfrac{\omega^{2}}{4a^{2}}}

Since c=f(0)=12πea2t2dt=212π0ea2t2dt=1a2πΓ(1/2)=1a2c = f(0) =\dfrac{1}{\sqrt{2 \pi}} \int_{- \infty} ^{\infty} e^{-a^2 t^2} dt = 2 \dfrac{1}{\sqrt{2 \pi}} \int_{0} ^{\infty} e^{-a^2 t^2} dt =\dfrac{1}{a \sqrt{2 \pi}} \Gamma(1/2) = \dfrac{1}{a\sqrt{2}}.

Therefore, f(ω)=1a2eω24a2f(\omega) =\dfrac{1}{a\sqrt{2}} e^{-\dfrac{\omega^{2}}{4a^{2}}}.

Surya Prakash - 5 years, 6 months ago

Problem 10:

Evaluate

0π/21(9tan2(x)+16)3dx\large \int_{0}^{\pi/2} \dfrac{1}{(9 \tan^2 (x) + 16)^3} \, dx

This problem has been solved by Pi Han Goh.

Surya Prakash - 5 years, 6 months ago

Log in to reply

Let II denote the value of the integral. And let 3tan(x)=4tan(y)3\tan (x) = 4\tan(y) , differentiate with respect to xx: 3sec2(x)=4sec2(y)(dydx)dx=4sec2(y)3sec2(x)dy=12sec2(y)9sec2(x)dy=12sec2(y)9tan2(x)+9dy=12sec2(y)16tan2(y)+9dy3\sec^2(x) = 4\sec^2(y) \left( \frac{dy}{dx} \right) \Rightarrow dx = \dfrac{4\sec^2(y)}{3\sec^2(x)} dy= \dfrac{12\sec^2(y)}{9\sec^2(x)} dy= \dfrac{12\sec^2(y)}{9\tan^2(x) + 9} dy= \dfrac{12\sec^2(y)}{16\tan^2(y) + 9} dy

And of course, 9tan2(x)+16=(3tan(x))2+16=16(tan2(y)+1)=16sec2(y)9\tan^2(x) + 16 = (3\tan(x))^2 + 16 = 16(\tan^2 (y) + 1) = 16\sec^2(y) .

When x=0x = 0, y=0y = 0; when xπ2x\to \frac\pi2^- , yπ2y \to \frac\pi2 ^- as well. The integral becomes

I=0π/21(16sec2(y))312sec2(y)16tan2(y)+9dy=121630π/21sec4(y)116tan2(y)+9dy=310240π/2cos6(y)16sin2(y)+9cos2(y)dy=310240π/2cos6(y)16(1cos2(y))+9cos2(y)dy=310240π/2cos6(y)167cos2(y)dy. \begin{aligned} I &=& \int_0^{\pi/2} \dfrac{1}{(16\sec^2(y))^3} \cdot \dfrac{12\sec^2(y)}{16\tan^2(y) + 9} \, dy \\ &=& \dfrac{12}{16^3} \int_0^{\pi/2} \dfrac1{\sec^4(y)} \cdot \dfrac1{16\tan^2 (y) + 9} \, dy \\ &=& \dfrac{3}{1024} \int_0^{\pi/2} \dfrac{\cos^6(y)}{16\sin^2 (y) + 9\cos^2(y)} \, dy \\ &=& \dfrac{3}{1024} \int_0^{\pi/2} \dfrac{\cos^6(y)}{16(1-\cos^2(y)) + 9\cos^2(y)} \, dy \\ &=& \dfrac{3}{1024} \int_0^{\pi/2} \dfrac{\cos^6(y)}{16 - 7\cos^2(y)} \, dy. \\ \end{aligned}

By long division, X3167X=17X21649X409634317X16256343 \dfrac{X^3}{16-7X} = -\dfrac17 X^2 - \dfrac{16}{49} X - \dfrac{4096}{343}\cdot \dfrac{1}{7X-16} - \dfrac{256}{343} . We contine from above,

10243I=0π/2[17cos4(y)1649cos2(y)409634317cos2(y)16256343]dy10243I=170π/2cos4(y)dy16490π/2cos2(y)dy81923430π/217(2cos2(y))32dy128π343=(173!!4!!π2)(16491!!2!!π2)81923430π/217cos(2y)25dy(128π343)=2643π5488+81923430π/21257cos(2y)dy. \begin{aligned} \dfrac{1024}3 I &=& \int_0^{\pi/2} \bigg [ -\dfrac17 \cos^4(y) - \dfrac{16}{49} \cos^2 (y) - \dfrac{4096}{343} \cdot \dfrac1{7\cos^2(y) - 16} - \dfrac{256}{343} \bigg ] \, dy \\ \dfrac{1024}3 I &=& -\dfrac17\int_0^{\pi/2} \cos^4(y) \, dy - \dfrac{16}{49} \int_0^{\pi/2} \cos^2 (y) \, dy - \dfrac{8192}{343} \cdot \int_0^{\pi/2} \dfrac1{7(2\cos^2(y)) - 32} \, dy - \dfrac{128\pi}{343} \\ &=& -\left(\dfrac17 \cdot \dfrac{3!!}{4!!} \cdot \dfrac\pi2\right) - \left(\dfrac{16}{49}\cdot \dfrac{1!!}{2!!} \cdot \dfrac{\pi}2\right) - \dfrac{8192}{343} \int_0^{\pi/2} \dfrac1{7\cos(2y) - 25} \, dy -\left(\dfrac{128\pi}{343} \right) \\ &=& -\dfrac{2643\pi}{5488} + \dfrac{8192}{343} \int_0^{\pi/2}\dfrac1{25 - 7\cos(2y)} \, dy. \\ \end{aligned}

For the final integral, let t=tan(y)dy=dtt2+1t = \tan(y) \Rightarrow dy = \dfrac{dt}{t^2 + 1} , and the integral becomes

0π/21257cos(2y)dy=01257(1t21+t2)dtt2+1=0t2+125+2t27t+7t2dtt2+1=0132t2+18dt=13201t2+(3/4)2dt=13243tan1(43t)0t=π48.\begin{aligned} \int_0^{\pi/2}\dfrac1{25 - 7\cos(2y)} \, dy &=& \int_0^\infty \dfrac{1}{25 - 7 \left( \frac{1-t^2}{1+t^2}\right) } \cdot \dfrac{dt}{t^2 + 1} \\ &=& \int_0^\infty \dfrac{t^2 + 1} {25 + 2t^2 - 7t + 7t^2} \cdot \dfrac{dt}{t^2 + 1} \\ &=& \int_0^\infty \dfrac{1}{32t^2 + 18} \, dt \\ &=& \dfrac1{32}\int_0^\infty \dfrac{1}{t^2 + (3/4)^2} \, dt \\ &=& \dfrac1{32} \cdot \left. \dfrac43 \tan^{-1} \left( \dfrac43 t \right) \right |_0^{t\to\infty} = \dfrac\pi{48}. \\ \end{aligned}

Simplify everything and we get the answer of I=263π56197120.00014702512653568645897079 I = \boxed{\dfrac{263 \pi}{5619712}} \approx 0.00014702512653568645897079 .


As Surya Prakesh has pointed out, there's an easier method.

It can be easily proved that 0π/21tan2(x)+pdx=π2(p+p)\int_{0}^{\pi /2} \dfrac{1}{\tan^2 (x ) + p} dx = \dfrac{\pi}{2(p+\sqrt{p})}. Then differentiate it twice w.r.t pp. Then take p=43p= \dfrac{4}{3}.

Pi Han Goh - 5 years, 6 months ago

Log in to reply

@Pi Han Goh Sir, shouldn't we put p=16/9 in the end........because, we have integrated (tanx) squared plus P.......

Aaghaz Mahajan - 2 years, 7 months ago

Problem 11:

Prove that

01ln(x+1)(x+1)(x2+1)dx=1192(36(ln2)2+12π(ln2)π2).\large \int_0^1 \dfrac{\ln(x+1)}{(x+1)(x^2 + 1)} \, dx = \dfrac1{192} (36 (\ln2)^2 + 12\pi (\ln2) - \pi^2 ).

This problem has been solved by Aditya Kumar.

Pi Han Goh - 5 years, 6 months ago

Log in to reply

Solution to Problem 11:

Pre-requisites:

I=aln(a+1)(a1)(a2+1)daApplyIntegralSubstitution:f(g(x))g(x)dx=f(u)du,u=g(x)u=aln(a+1):u=aa+1+ln(a+1)da,da=1aa+1+ln(a+1)duI=u(a1)(a2+1)1aa+1+ln(a+1)duI=1(a1)(a2+1)(aa+1+ln(a+1))u22OnsubstitutionI=a2ln2(a+1)2(a1)(a2+1)(aa+1+ln(a+1))I=\int \frac { -a\ln { \left( a+1 \right) } }{ \left( a-1 \right) \left( a^{ 2 }+1 \right) } da\\ { Apply\: Integral\: Substitution }:\quad \\ \int f\left( g\left( x \right) \right) \cdot g^{ ' }\left( x \right) dx=\int f\left( u \right) du,\: \quad u=g\left( x \right) \\ u=a\ln { \left( a+1 \right) } :\\ u=\frac { a }{ a+1 } +\ln { \left( a+1 \right) } da,\\ da=\frac { 1 }{ \frac { a }{ a+1 } +\ln { \left( a+1 \right) } } du\\ I=-\int \frac { u }{ \left( a-1 \right) \left( a^{ 2 }+1 \right) } \frac { 1 }{ \frac { a }{ a+1 } +\ln { \left( a+1 \right) } } du\\ I=-\frac { 1 }{ \left( a-1 \right) \left( a^{ 2 }+1 \right) \left( \frac { a }{ a+1 } +\ln { \left( a+1 \right) } \right) } \frac { u^{ 2 } }{ 2 } \\ On\quad substitution\quad \quad \\ I=-\frac { a^{ 2 }\ln { ^{ 2 } } \left( a+1 \right) }{ 2\left( a-1 \right) \left( a^{ 2 }+1 \right) \left( \frac { a }{ a+1 } +\ln { \left( a+1 \right) } \right) }

I(a)=01ln(1+ax)(1+x)(1+x2)dxI(a)=01x(1+x)(1+ax)(1+x2)dxOnsplittingaspartialfractionsandsolving,I(a)=aln(a)(a1)(a2+1)+ln22(a+1)+ln2(1a)4(a2+1)I(a)=01(aln(a)(a1)(a2+1)+ln22(a+1)+ln2(1a)4(a2+1))daNowonsolvingandsubstitutinga=1,wegetI=1192(36(ln2)2+12π(ln2)π2)I\left( a \right) =\int _{ 0 }^{ 1 }{ \frac { ln\left( 1+ax \right) }{ \left( 1+x \right) \left( 1+{ x }^{ 2 } \right) } dx } \\ { I }^{ ' }\left( a \right) =\int _{ 0 }^{ 1 }{ \frac { x }{ \left( 1+x \right) \left( 1+ax \right) \left( 1+{ x }^{ 2 } \right) } dx } \\ On\quad splitting\quad as\quad partial\quad fractions\quad and\quad solving,\\ { I }^{ ' }\left( a \right) =\frac { -aln(a) }{ (a-1)({ a }^{ 2 }+1) } +\frac { ln2 }{ 2(a+1) } +\frac { ln2(1-a) }{ 4({ a }^{ 2 }+1) } \\ I\left( a \right) =\int _{ 0 }^{ 1 }{ \left( \frac { -aln(a) }{ (a-1)({ a }^{ 2 }+1) } +\frac { ln2 }{ 2(a+1) } +\frac { ln2(1-a) }{ 4({ a }^{ 2 }+1) } \right) da } \\ Now\quad on\quad solving\quad and\quad substituting\quad a=1,\quad we\quad get\\ I=\frac { 1 }{ 192 } \left( 36{ \left( ln2 \right) }^{ 2 }+12\pi \left( ln2 \right) -{ \pi }^{ 2 } \right)

Aditya Kumar - 5 years, 6 months ago

Problem 12:

Prove or disprove:

If aa is not a natural number, then

0π/2(sinθ)a1(cosθ)2tasin(2tθ)dθ=πΓ(2ta)2sin(πa2)Γ(2t)Γ(a). \large \int_0^{\pi /2} (\sin \theta)^{a-1} (\cos \theta)^{2t-a}\sin(2t\theta) \, d\theta = \dfrac{\pi \Gamma(2t-a)}{2\sin\left( \frac{\pi a}2\right) \Gamma(2t) \Gamma(-a) }.

This problem has been solved by Tanishq Varshney.

Aditya Kumar - 5 years, 6 months ago

Log in to reply

Solution to Problem 12:\large{Solution ~to~Problem ~12:}

f(x)=sin(2tarctanx)x(1+x)t=k=0Γ(2t+2k+1)Γ(k+1)Γ(2t)Γ(2k+2).(x)kk!\Large{f(x)=\frac{\sin (2 t \arctan \sqrt{x})}{\sqrt{x} (1+x)^t}=\displaystyle \sum^{\infty}_{k=0} \frac{\Gamma(2t+2k+1) \Gamma(k+1)}{\Gamma(2t) \Gamma(2k+2)}.\frac {(-x)^{k}}{k!}}

Using Ramanujan master theorem

0xs1f(x)=Γ(s)ϕ(s)\Large{\displaystyle \int^{\infty}_{0} x^{s-1} f(x)=\Gamma(s) \phi(-s)}

where, ϕ(k)=Γ(2t+2k+1)Γ(k+1)Γ(2t)Γ(2k+2)\phi(k)=\frac{\Gamma(2t+2k+1) \Gamma(k+1)}{\Gamma(2t) \Gamma(2k+2)}

xtan2θx \rightarrow \tan^2 \theta

0π2sina1θ(cosθ)2tacos(2tθ)dθ=πΓ(2ta)2sin(πa2)Γ(2t)Γ(a)\Large{\displaystyle \int^{\frac{\pi}{2}}_{0} sin^{a-1} \theta (\cos \theta )^{2t-a} \cos (2t \theta) d \theta =\frac{\pi \Gamma(2t-a)}{2 \sin \left( \frac{\pi a}{2} \right) \Gamma(2t) \Gamma(-a)}}

Tanishq Varshney - 5 years, 6 months ago

Problem 14:

Evaluate 011xlnx(x+x2+x22+x23+)dx.\Large {{\displaystyle \int^{1}_{0} \frac{1-x}{\ln x} \left(x+x^2+x^{2^2}+x^{2^3}+\cdots \right) \, dx}}.

This problem has been solved by Surya Prakash.

Tanishq Varshney - 5 years, 6 months ago

Log in to reply

01(1x)ln(x)(x+x2+x22+x23+)=n=001x2nx2n+1ln(x)dx\int_{0}^{1} \dfrac{(1-x)}{\ln (x)} ( x + x^2 + x^{2^2} + x^{2^3} + \ldots ) = \sum_{n=0}^{\infty} \int_{0}^{1} \dfrac{x^{2^n} - x^{2^n + 1}}{\ln (x)} dx

Let f(a)=01xalnxdxf(a) = \int_{0}^{1} \dfrac{x^a}{\ln x} dx. Differentiate it w.r.t aa. We get

f(a)=01xadx=1a+1f'(a) = \int_{0}^{1} x^a dx = \dfrac{1}{a+1}

So,

01x2nx2n+1ln(x)dx=2n+12nf(a)da=ln(2n+12n+2)\int_{0}^{1} \dfrac{x^{2^n} - x^{2^n + 1}}{\ln(x)} dx = \int_{2^n+1}^{2^n} f'(a) da = \ln \left(\dfrac{2^n + 1}{2^n + 2} \right)

01x2nx2n+1ln(x)dx=ln(2n+1)ln(2n+2)\int_{0}^{1} \dfrac{x^{2^n} - x^{2^n + 1}}{\ln(x)} dx = \ln (2^n +1) - \ln (2^n +2)

n=001x2nx2n+1ln(x)dx=limNn=0Nln(2n+1)ln(2n+2)=limNn=0Nln(2n+1)ln(2)ln(2n1+1)\sum_{n=0}^{\infty} \int_{0}^{1} \dfrac{x^{2^n} - x^{2^n + 1}}{\ln(x)} dx = \lim_{N \rightarrow \infty} \sum_{n=0}^{N} \ln (2^n +1) - \ln (2^n +2) = \lim_{N \rightarrow \infty} \sum_{n=0}^{N} \ln (2^n +1) -\ln (2) - \ln (2^{n-1} +1)

So, we have got a telescoping series which sums up to

=limN(N+1)ln(2)ln(3/2)+ln(2N+1)=ln(3)= \lim_{N \rightarrow \infty} -(N+1)\ln(2) - \ln(3/2) + \ln (2^N + 1) = -\ln(3)

Surya Prakash - 5 years, 6 months ago

Problem 15:

Find the closed form for I(x)I(x) , where I(x)=π/2π/2cos(xtan(θ))dθ.I(x) = \large \int_{-\pi /2}^{\pi /2} \cos(x\tan(\theta)) \, d\theta .

This problem has been solved by Tanishq Varshney via contour integration. And Surya Prakesh posted his intended solution.

Surya Prakash - 5 years, 6 months ago

Log in to reply

Another interesting elementary solution :

Lemma :

0cosmxx2+a2dx=π2aeam\large{\displaystyle \int_{0}^{\infty} \frac{\cos mx}{x^2+a^2} \mathrm{d} x=\frac{\pi}{2a} e^{-am}}

Proof :

\text{I}(m) = \displaystyle \int_{0}^{\infty} \dfrac{\cos mx}{x^2+a^2} \mathrm{d} x \tag{1}

Using Integration by parts,

I(m)=02xsinmxm(x2+a2)2dx \text{I}(m) = \displaystyle \int_{0}^{\infty} \dfrac{2x\sin mx}{m(x^2+a^2)^2} \mathrm{d} x

\implies m \cdot \text{I}(m) = \displaystyle \int_{0}^{\infty} \dfrac{2x \sin mx}{(x^2+a^2)^2} \mathrm{d} x \tag {2}

Partially Differentiating (2)(2) w.r.t. mm, we have,

I+mmI=02x2cos(mx)(x2+a2)2dx=02cos(mx)(x2+a2)dx2a20cosmx(x2+a2)2dx\text{I} + m\dfrac{\partial}{\partial m} \text{I} = \displaystyle \int_{0}^{\infty} \dfrac {2x^2\cos(mx)}{(x^2+a^2)^2} \mathrm{d}x = \displaystyle \int_{0}^{\infty} \dfrac {2\cos(mx)}{(x^2+a^2)} \mathrm{d}x - 2a^2\displaystyle \int_{0}^{\infty}\dfrac{\cos mx}{(x^2+a^2)^2} \mathrm{d}x

\implies a\dfrac{\partial}{\partial m} \text{I} = \text{I} - 2b^2\displaystyle \int_{0}^{\infty}\dfrac{\cos mx}{(x^2+a^2)^2} \mathrm{d}x \tag{3}

Again, partially differentiating (3)(3) w.r.t. mm, we have,

m2m2I=a202xsinmx(x2+a2)2dxm\dfrac{\partial^2}{\partial m^2} \text{I} = a^2 \displaystyle \int_{0}^{\infty} \dfrac{2x \sin mx}{(x^2+a^2)^2} \mathrm{d} x

\implies \dfrac{\partial^2}{\partial m^2} \text{I} = a^2 \text{I} \tag{*} (from (2)(2))

Note that I(0)=π2aI(0) = \dfrac{\pi}{2a} and I()=0I(\infty) = 0

Now, solving ()(*), we have,

I=π2aeamI = \dfrac{\pi}{2a} e^{-am}

Ishan Singh - 5 years, 6 months ago

Log in to reply

Oh, I suddenly noticed I reproduced your method independently after several days and just didn't noticed that it is already posted here.

Ronak Agarwal - 5 years, 5 months ago

Solution to Problem 15Problem ~15

Lemma

0cosmxx2+a2dx=π2aeam\large{\displaystyle \int^{\infty}_{0} \frac{\cos mx}{x^2+a^2} dx=\frac{\pi}{2a} e^{-am}}

Proof\text{Proof}

Consider the integral Cf(z)dz\large{\displaystyle \int _{ C } f\left( z \right) dz} where f(z)=eimzz2+a2\large{f(z)=\frac{e^{imz}}{z^2+a^2}} taken round the closed contour CC consisting of the upper half of a large circle z=R|z|=R and the real axis from R-R to RR.

Poles of f(z)f(z) are given by

z2+a2=0z=±aiz^2+a^2=0 \rightarrow z=\pm ai

The only pole which lies within the contour is at z=aiz=ai. The residue of f(z)f(z) at z=aiz=ai

=limzai(zai)eimzz2+a2=limzaieimzz+ai=eam2ai\large{= \displaystyle \lim_{z \to ai} \frac{(z-ai)e^{imz}}{z^2+a^2}= \displaystyle \lim_{z \to ai} \frac{e^{imz}}{z+ai}=\frac{e^{-am}}{2ai}}

Hence by cauchy residue theorem

Cf(z)dz=2πi×sum of residues\large{\displaystyle \int _{ C } f\left( z \right) dz=2 \pi i \times \text{sum of residues}}

Cf(z)dz=2πi×eam2ai\large{\displaystyle \int _{ C } f\left( z \right) dz=2 \pi i \times \frac{e^{-am}}{2ai} }

Ceimzz2+a2dzRReimxx2+a2dx=πeama\large{\displaystyle \int _{ C }\frac{e^{imz}}{z^2+a^2} dz \rightarrow \displaystyle \int^{R} _{ -R }\frac{e^{imx}}{x^2+a^2} dx= \frac{ \pi e^{-am}}{a}}

Equating Real parts

cosmxx2+a2dx=πeama\large{\displaystyle \int^{\infty} _{ -\infty }\frac{\cos mx}{x^2+a^2} dx= \frac{ \pi e^{-am}}{a}}

0cosmxx2+a2dx=πeam2a\large{\Longrightarrow \displaystyle \int^{\infty} _{ 0 }\frac{\cos mx}{x^2+a^2} dx= \frac{ \pi e^{-am}}{2a}}

Now in the integral put tanθ=t\tan \theta =t

cosxtt2+1dt=πex\large{\displaystyle \int^{\infty} _{ - \infty }\frac{\cos xt}{t^2+1} dt=\pi e^{-x}}

I would like to see the solution which doesn't involve contour!!

Tanishq Varshney - 5 years, 6 months ago

Lemma: cos(xt)1+t2dt=eixt1+t2dt\int_{-\infty}^{\infty} \dfrac{\cos (xt)}{1+t^2} dt = \int_{-\infty}^{\infty} \dfrac{e^{-ixt}}{1+t^2} dt

Try to prove this yourselves.

Consider,

f(t)=exeixtdx=21+t2f(t) = \int_{-\infty}^{\infty} e^{-|x|} e^{ixt}dx = \dfrac{2}{1+t^2}

Above integral is easy to evaluate.

So,

f(t)=21+t2f(t)=\dfrac{2}{1+t^2}

Now by Fourier inversion formula,

ex=12πf(t)eixtdte^{-|x|} = \dfrac{1}{2\pi} \int_{-\infty}^{\infty} f(t) e^{-ixt} dt

So we get that

cos(xt)1+t2dt=eixt1+t2dt=πex\int_{-\infty}^{\infty} \dfrac{\cos (xt)}{1+t^2} dt = \int_{-\infty}^{\infty} \dfrac{e^{-ixt}}{1+t^2} dt = \pi e^{-|x|}

Surya Prakash - 5 years, 6 months ago

Problem 7:

For 0<v<10 < v < 1 independent of xx, prove that: 0[xv3(γx+logΓ(1+x))]dx=πsin(π(v))ζ(2v)2v.\large \int _{ 0 }^{ \infty }{ \left[ { x }^{ v-3 }\left( \gamma x+\log\Gamma \left( 1+x \right) \right) \right] \, dx } =\frac { \pi }{ \sin\left( \pi \left( v \right) \right) }\cdot \frac { \zeta \left( 2-v \right) }{ 2-v }.

Due to time constraint, the author decided to post the solution.

Aditya Kumar - 5 years, 6 months ago

Log in to reply

Solution to Problem 7:

Lemma:

Γ(x)=eγxxn=1(1+xn)1exn\Gamma \left( x \right) =\frac { { e }^{ -\gamma x } }{ x } \prod _{ n=1 }^{ \infty }{ { \left( 1+\frac { x }{ n } \right) }^{ -1 }{ e }^{ \frac { x }{ n } } }

Proof:

Γ(x)=limn(nxxi=1n(ix+i))Γ(x)=limn(nxxi=1n(1+xi)1)Γ(x)=limn(exlog(n)xi=1n(1+xi)1)Γ(x)=limn(ei=1(xn)i=1(xn)+xlog(n)xi=1n(1+xi)1)But,γ=limn(i=1(1n)log(n))Γ(x)=limn(ei=1(xn)γxxi=1n(1+xi)1)Γ(x)=limn(eγxxi=1ne(xn)(1+xi)1)\Gamma \left( x \right) =\lim _{ n\rightarrow \infty }{ \left( \frac { { n }^{ x } }{ x } \prod _{ i=1 }^{ n }{ \left( \frac { i }{ x+i } \right) } \right) } \\ \Gamma \left( x \right) =\lim _{ n\rightarrow \infty }{ \left( \frac { { n }^{ x } }{ x } \prod _{ i=1 }^{ n }{ { \left( 1+\frac { x }{ i } \right) }^{ -1 } } \right) } \\ \Gamma \left( x \right) =\lim _{ n\rightarrow \infty }{ \left( \frac { { e }^{ xlog\left( n \right) } }{ x } \prod _{ i=1 }^{ n }{ { \left( 1+\frac { x }{ i } \right) }^{ -1 } } \right) } \\ \Gamma \left( x \right) =\lim _{ n\rightarrow \infty }{ \left( \frac { { e }^{ \sum _{ i=1 }^{ \infty }{ \left( \frac { x }{ n } \right) -\sum _{ i=1 }^{ \infty }{ \left( \frac { x }{ n } \right) } + } xlog\left( n \right) } }{ x } \prod _{ i=1 }^{ n }{ { \left( 1+\frac { x }{ i } \right) }^{ -1 } } \right) } \\ But,\quad \gamma =\lim _{ n\rightarrow \infty }{ \left( \sum _{ i=1 }^{ \infty }{ \left( \frac { 1 }{ n } \right) } -log\left( n \right) \right) } \\ \Gamma \left( x \right) =\lim _{ n\rightarrow \infty }{ \left( \frac { { e }^{ \sum _{ i=1 }^{ \infty }{ \left( \frac { x }{ n } \right) -\gamma x } } }{ x } \prod _{ i=1 }^{ n }{ { \left( 1+\frac { x }{ i } \right) }^{ -1 } } \right) } \\ \Gamma \left( x \right) =\lim _{ n\rightarrow \infty }{ \left( \frac { { e }^{ -\gamma x } }{ x } \prod _{ i=1 }^{ n }{ { { e }^{ \left( \frac { x }{ n } \right) }\left( 1+\frac { x }{ i } \right) }^{ -1 } } \right) }

Now taking log on both sides and replacing x by x+1, we get

log(Γ(1+x))=γx+k=2(ζ(k)k(x)k)\log { \left( \Gamma \left( 1+x \right) \right) } =-\gamma x+\sum _{ k=2 }^{ \infty }{ \left( \frac { \zeta \left( k \right) }{ k } { \left( -x \right) }^{ k } \right) }

Now plugging these values into the question and by using Ramanujan's master theorem, we get

0[xv1(γx+logΓ(1+x))x2]dx=πsin(π(v))ζ(2v)2v\int _{ 0 }^{ \infty }{ \left[ \frac { { x }^{ v-1 }\left( \gamma x+\log \Gamma \left( 1+x \right) \right) }{ { x }^{ 2 } } \right] \, dx } =\frac { \pi }{ \sin \left( \pi \left( v \right) \right) } \cdot \frac { \zeta \left( 2-v \right) }{ 2-v }

Aditya Kumar - 5 years, 6 months ago

Alternate Solution to Problem 7

Lemma : 0xnex1dx=Γ(n+1)ζ(n+1) \displaystyle \int _{ 0 }^{ \infty }{ \frac { { x }^{ n } }{ { e }^{ x }-1 } dx } =\Gamma (n+1)\zeta (n+1)

Proof :I=0xnex1dx=0xnex1exdx=0xnr=1erxdx=r=10xnerxdx \displaystyle I = \int _{ 0 }^{ \infty }{ \frac { { x }^{ n } }{ { e }^{ x }-1 } dx } =\int _{ 0 }^{ \infty }{ \frac { { x }^{ n }{ e }^{ -x } }{ 1-{ e }^{ -x } } dx } =\int _{ 0 }^{ \infty }{ { x }^{ n }\sum _{ r=1 }^{ \infty }{ { e }^{ -rx } } dx } =\sum _{ r=1 }^{ \infty }{ \int _{ 0 }^{ \infty }{ { x }^{ n }{ e }^{ -rx }dx } }

Note that I have used the series expansion of y1y \dfrac{y}{1-y} and interchanged summation and integral.

I=r=10xnerxdx=r=1Γ(n+1)rn+1=Γ(n+1)ζ(n+1) \displaystyle I = \sum _{ r=1 }^{ \infty }{ \int _{ 0 }^{ \infty }{ { x }^{ n }{ e }^{ -rx }dx } } =\quad \sum _{ r=1 }^{ \infty }{ \frac { \Gamma (n+1) }{ { r }^{ n+1 } } } =\Gamma (n+1)\zeta (n+1)

I have here used the definitions of gamma function and the zeta function.

Now back to our problem.

Integrate it by parts to get :

I=xv2v2(γx+logΓ(1+x))0+12v0xv2(γ+ψ(1+x))dx=12v0xv2(γ+ψ(1+x))dx \displaystyle I = \frac { { x }^{ v-2 } }{ v-2 } (\gamma x+\log {\Gamma (1+x) } ){ | }_{ 0 }^{ \infty }+\frac { 1 }{ 2-v } \int _{ 0 }^{ \infty }{ { x }^{ v-2 }(\gamma +\psi (1+x))dx } = \frac { 1 }{ 2-v } \int _{ 0 }^{ \infty }{ { x }^{ v-2 }(\gamma +\psi (1+x))dx }

Using the definition that γ+ψ(1+x)=011yx1ydy \displaystyle \gamma +\psi (1+x)=\int _{ 0 }^{ 1 }{ \frac { 1-{ y }^{ x } }{ 1-y } dy } , we get :

I=12v001xv2(1yx1y)dxdy \displaystyle I = \frac { 1 }{ 2-v } \int _{ 0 }^{ \infty }{ \int _{ 0 }^{ 1 }{ { x }^{ v-2 }(\frac { 1-{ y }^{ x } }{ 1-y } )dxdy } }

Changing the order of integration we have :

I=12v0111y0xv2(1yx)dxdy \displaystyle I = \frac { 1 }{ 2-v } \int _{ 0 }^{ 1 }{ \frac { 1 }{ 1-y } \int _{ 0 }^{ \infty }{ { x }^{ v-2 }(1-{ y }^{ x })dxdy } }

Integrating it by parts we have :

I=12v(01dy1y(xv1(1yx)v10+ln(y)v10xv1yxdx)) \displaystyle I = \frac { 1 }{ 2-v } (\int _{ 0 }^{ 1 }{ \frac { dy }{ 1-y } (\frac { { x }^{ v-1 }(1-{ y }^{ x }) }{ v-1 } { | }_{ 0 }^{ \infty }+\frac { \ln(y) }{ v-1 } \int _{ 0 }^{ \infty }{ { x }^{ v-1 }{ y }^{ x }dx } ) } )

I=1(v1)(2v)(01ln(y)dy1y(0xv1yxdx))\displaystyle I = \frac { 1 }{ (v-1)(2-v) } (\int _{ 0 }^{ 1 }{ \frac { \ln(y) dy }{ 1-y } (\int _{ 0 }^{ \infty }{ { x }^{ v-1 }{ y }^{ x }dx } ) } )

I=1(v1)(2v)(01ln(y)dy(1y)(0xv1ex(ln(y))dx)) \displaystyle I = \frac { 1 }{ (v-1)(2-v) } (\int _{ 0 }^{ 1 }{ \frac { \ln(y) dy }{ (1-y) } (\int _{ 0 }^{ \infty }{ { x }^{ v-1 }{ e }^{ -x(-\ln { (y)) } }dx } ) } )

I=1(v1)(2v)(01ln(y)dy(1y)(ln(y))v(0xv1exdx)) \displaystyle \Rightarrow I = \frac { 1 }{ (v-1)(2-v) } (\int _{ 0 }^{ 1 }{ \frac { \ln { (y) } dy }{ (1-y){ (-\ln { (y)) } }^{ v } } (\int _{ 0 }^{ \infty }{ { x }^{ v-1 }{ e }^{ -x }dx } ) } )

I=Γ(v)(v1)(2v)(01ln(y)dy(1y)(ln(y))v) \displaystyle \Rightarrow I = \frac { \Gamma (v) }{ (v-1)(2-v) } (\int _{ 0 }^{ 1 }{ \frac { \ln { (y) } dy }{ (1-y){ (-\ln { (y)) } }^{ v } } } )

Put y=exy={e}^{-x}

I=Γ(v)(1v)(2v)(0x1vdxex1) \displaystyle \Rightarrow I = \frac { \Gamma (v) }{ (1-v)(2-v) } (\int _{ 0 }^{ \infty }{ \frac { { x }^{ 1-v }dx }{ { e }^{ x }-1 } } )

Using Lemma we have :

I=Γ(v)Γ(2v)ζ(2v)(1v)(2v)=Γ(v)Γ(1v)ζ(2v)2v \displaystyle I = \frac { \Gamma (v)\Gamma (2-v)\zeta (2-v) }{ (1-v)(2-v) } =\frac { \Gamma (v)\Gamma (1-v)\zeta (2-v) }{ 2-v }

Using gamma reflection formula it becomes :

I=πζ(2v)sin(πv)(2v) \displaystyle I = \frac { \pi \zeta (2-v) }{ \sin { (\pi v) } (2-v) }

Hence Proved.

Ronak Agarwal - 5 years, 5 months ago

Problem 19:

Prove that

0π3ln2(sinxsin(π3+x))dx=581π3.\large{\displaystyle \int^{\frac{\pi}{3}}_{0} \ln^{2} \left( \frac{\sin x}{\sin \left(\frac{\pi}{3}+x \right)} \right) \, dx=\frac{5}{81} \pi^{3}}.

This problem has been solved by Surya Prakesh.

Tanishq Varshney - 5 years, 6 months ago

Log in to reply

Alternate Solution to problem 19 :

Lemma: 0xm1ln2(x)1+xndx=(πn)3(csc(mπn)cot2(mπn)+csc3(mπn)) \displaystyle \int _{ 0 }^{ \infty }{ \frac { { x }^{ m-1 }{ ln }^{ 2 }(x) }{ 1+{ x }^{ n } } dx } ={ \left(\frac { \pi }{ n } \right) }^{ 3 }\left(\csc { \left(\frac { m\pi }{ n } \right) } \cot ^{ 2 }{ \left(\frac { m\pi }{ n } \right) } +\csc ^{ 3 }{ \left(\frac { m\pi }{ n } \right) } \right)\quad

Proof : We begin with identity I=0dx1+xn=πncsc(πn)\displaystyle I = \int _{ 0 }^{ \infty }{ \frac { dx }{ 1+{ x }^{ n } } } =\frac { \pi }{ n } \csc { (\frac { \pi }{ n } ) }

To prove this put y=11+xn \displaystyle y = \dfrac{1}{1+{x}^{n}} to get :

I=1n01y1/n(1y)1/n1dy\displaystyle I =\dfrac{1}{n} \int _{ 0 }^{ 1 }{ { y }^{ -1/n }{ (1-y) }^{ 1/n-1 }dy }

Use definition of beta function and euler reflection formula to get :

I=1n01y1/n(1y)1/n1dy=1nΓ(1/n)Γ(11/n)Γ(1)=πncsc(πn)\displaystyle I = \dfrac{1}{n}\int _{ 0 }^{ 1 }{ { y }^{ -1/n }{ (1-y) }^{ 1/n-1 }dy } =\dfrac{1}{n}\frac { \Gamma (1/n)\Gamma (1-1/n) }{ \Gamma (1) } =\frac { \pi }{ n } \csc { (\frac { \pi }{ n } ) }

Now consider the integral J(m)=0xm1dx1+xn \displaystyle J(m) = \int _{ 0 }^{ \infty }{ \frac { { x }^{ m-1 }dx }{ 1+{ x }^{ n } } }

Put xm=y{x}^{m}=y to get :

J(m)=1m0dy1+yn/m \displaystyle J(m) = \frac { 1 }{ m } \int _{ 0 }^{ \infty }{ \frac { dy }{ 1+{ y }^{ n/m } } }

Use the identity to get :

J(m)=πncsc(mπn) \displaystyle J(m) = \frac { \pi }{ n } \csc { (\frac { m\pi }{ n } ) }

Differentiate it two times with respect to m m to prove the lemma :

My initial steps are the same as surya prakash and I directly get till :

I=3201(1+y)ln2(y)1+y3dy \displaystyle I = \frac { \sqrt { 3 } }{ 2 } \int _{ 0 }^{ 1 }{ \frac { (1+y){ ln }^{ 2 }(y) }{ 1+{ y }^{ 3 } } dy }

Put y=1x \displaystyle y=\dfrac{1}{x} to get :

I=321(1+x)ln2(x)1+x3dx \displaystyle I = \frac { \sqrt { 3 } }{ 2 } \int _{ 1 }^{ \infty }{ \frac { (1+x){ ln }^{ 2 }(x) }{ 1+{ x }^{ 3 } } dx }

Adding these two forms we have :

I=340(1+x)ln2(x)1+x3dx \displaystyle I = \frac { \sqrt { 3 } }{ 4 } \int _{ 0 }^{ \infty }{ \frac { (1+x){ ln }^{ 2 }(x) }{ 1+{ x }^{ 3 } } dx }

I=34(0ln2(x)1+x3dx+0xln2(x)1+x3dx) \displaystyle I = \frac { \sqrt { 3 } }{ 4 } (\int _{ 0 }^{ \infty }{ \frac { { ln }^{ 2 }(x) }{ 1+{ x }^{ 3 } } dx } +\int _{ 0 }^{ \infty }{ \frac { x{ ln }^{ 2 }(x) }{ 1+{ x }^{ 3 } } dx } )

Use the lemma to get :

I=34((π3)3(23.13+833)+(π3)3(23.13+833))=5π381 \displaystyle I = \frac { \sqrt { 3 } }{ 4 } ({ (\frac { \pi }{ 3 } ) }^{ 3 }(\frac { 2 }{ \sqrt { 3 } } .\frac { 1 }{ 3 } +\frac { 8 }{ 3\sqrt { 3 } } )+{ (\frac { \pi }{ 3 } ) }^{ 3 }(\frac { 2 }{ \sqrt { 3 } } .\frac { 1 }{ 3 } +\frac { 8 }{ 3\sqrt { 3 } } ))=\frac { 5{ \pi }^{ 3 } }{ 81 }

Ronak Agarwal - 5 years, 5 months ago

Lemma: 01xmlnn(x)dx=(1)nn!(m+1)n+1\int_{0}^{1} x^{m} \ln ^{n} (x) dx = \dfrac{(-1)^n n!}{(m+1)^{n+1}}

Let y=sinxsin(x+π/3)    tanx=3y2y    dx=321y2y+1dyy=\dfrac{\sin x}{\sin \left(x+\pi/3 \right)} \implies \tan x = \dfrac{\sqrt{3}y}{2-y} \implies dx = \dfrac{\sqrt{3}}{2} \dfrac{1}{y^2 - y +1}dy .

So,

0π/3ln2(sinxsin(π3+x))dx=3201ln2(y)y2y+1dy=3201(1+y)ln2(y)1+y3dy\begin{aligned} \int_{0}^{\pi/3} \ln ^2 \left(\dfrac{\sin x}{\sin \left(\dfrac{\pi}{3} + x \right) } \right) dx &= \dfrac{\sqrt{3}}{2} \int_{0}^{1} \dfrac{\ln ^2 (y)}{y^2 - y +1} dy \\ &= \dfrac{\sqrt{3}}{2} \int_{0}^{1} \dfrac{(1+y)\ln ^2 (y)}{1+y^3} dy \end{aligned}

Since, 0<y<10 < y < 1, we can use the expansion 11+y3=1y3+y6+=k=0(1)ky3k\dfrac{1}{1+y^3} = 1-y^3 + y^6 + \ldots = \sum_{k=0}^{\infty} (-1)^k y^{3k}.

So, the integral becomes,

01(1+y)ln2(y)1+y3dy=01(1+y)ln2(y)k=0(1)ky3kdy=k=0(1)k[01y3kln2(y)dy+01y3k+1ln2(y)dy]=2k=0(1)k(3k+1)3+2k=0(1)k(3k+2)3\begin{aligned} \int_{0}^{1} \dfrac{(1+y)\ln ^2 (y)}{1+y^3} dy &= \int_{0}^{1} (1+y)\ln ^2 (y) \sum_{k=0}^{\infty} (-1)^k y^{3k} dy \\ &= \sum_{k=0}^{\infty} (-1)^k \Bigr[ \int_{0}^{1} y^{3k} \ln^2 (y) dy + \int_{0}^{1} y^{3k+1} \ln^2 (y) dy \Bigr] \\ &= 2 \sum_{k=0}^{\infty} \dfrac{(-1)^k}{(3k+1)^3} + 2 \sum_{k=0}^{\infty} \dfrac{(-1)^k}{(3k+2)^3} \end{aligned}

So,

0π/3ln2(sinxsin(π3+x))dx=3[k=0(1)k(3k+1)3+k=0(1)k(3k+2)3]\int_{0}^{\pi/3} \ln ^2 \left(\dfrac{\sin x}{\sin \left(\dfrac{\pi}{3} + x \right) } \right) dx = \sqrt{3} \Bigr[ \sum_{k=0}^{\infty} \dfrac{(-1)^k}{(3k+1)^3} + \sum_{k=0}^{\infty} \dfrac{(-1)^k}{(3k+2)^3} \Bigr]

Here comes the tricky part,

Consider the series (0<q<1)(0< q< 1)

1q31(1+q)3+1(2+q)3\dfrac{1}{q^3} - \dfrac{1}{(1+q)^3} + \dfrac{1}{(2+q)^3} - \ldots

It is easy to prove that above series is equal to 18(ζ(3,q2)ζ(3,q+12))\dfrac{1}{8} \left( \zeta \left(3, \dfrac{q}{2} \right) - \zeta \left(3, \dfrac{q+1}{2} \right) \right), where ζ(s,q)\zeta(s,q) is Hurwitz Zeta Function.

Using the relation between Polygamma function and Hurwitz Zeta Function, We get

1q31(1+q)3+1(2+q)3=116(ψ(2)(q+12)ψ(2)(q2))\dfrac{1}{q^3} - \dfrac{1}{(1+q)^3} + \dfrac{1}{(2+q)^3} - \ldots = \dfrac{1}{16} \left(\psi_{(2)} \left(\dfrac{q+1}{2} \right) - \psi_{(2)} \left(\dfrac{q}{2} \right) \right)

So, finally

k=0(1)k(3k+1)3+k=0(1)k(3k+2)3=127(k=0(1)k(k+1/3)3+k=0(1)k(k+2/3)3)=127116(ψ(2)(16)ψ(2)(23)+ψ(2)(13)ψ(2)(56))=53π3243\begin{aligned} \sum_{k=0}^{\infty} \dfrac{(-1)^k}{(3k+1)^3} + \sum_{k=0}^{\infty} \dfrac{(-1)^k}{(3k+2)^3} &= \dfrac{1}{27} \left( \sum_{k=0}^{\infty} \dfrac{(-1)^k}{(k+1/3)^3} + \sum_{k=0}^{\infty} \dfrac{(-1)^k}{(k+2/3)^3} \right) \\ &= \dfrac{1}{27} \dfrac{1}{16} \left(\psi_{(2)} \left(\dfrac{1}{6}\right)-\psi_{(2)}\left(\dfrac{2}{3}\right)+\psi_{(2)}\left(\dfrac{1}{3}\right)-\psi_{(2)}\left(\dfrac{5}{6}\right) \right) = \dfrac{5 \sqrt{3} \pi^3}{243} \end{aligned}

Above final calculation is done using Euler's Reflection Formula for polygamma function

So, finally

0π/3ln2(sinxsin(π3+x))dx=3[k=0(1)k(3k+1)3+k=0(1)k(3k+2)3]=353π3243=5π381 \int_{0}^{\pi/3} \ln ^2 \left(\dfrac{\sin x}{\sin \left(\dfrac{\pi}{3} + x \right) } \right) dx = \sqrt{3} \Bigr[ \sum_{k=0}^{\infty} \dfrac{(-1)^k}{(3k+1)^3} + \sum_{k=0}^{\infty} \dfrac{(-1)^k}{(3k+2)^3} \Bigr] = \sqrt{3} \dfrac{5 \sqrt{3}\pi^3}{243} = \dfrac{5 \pi^3}{81}

Sorry for missing the calculation part, As it became too lengthy to type here.

Notify me if there are any typing mistakes.

Surya Prakash - 5 years, 6 months ago

Log in to reply

Thanks for everyone. Because of this competition and you guys I learnt a lot of integrations skills and develop my integration skills so fast. I thank everyone. @Pi Han Goh @Aditya Kumar @Tanishq Varshney @Sudeep Salgia @Abhishek Bakshi

Surya Prakash - 5 years, 6 months ago

Problem 25

Prove that

0π2ln(sinx)ln(cosx)tanxdx=18ζ(3).\large{\displaystyle \int^{\frac{\pi}{2}}_{0}\frac{\ln(\sin x) \ln(\cos x)}{\tan x} dx=\frac{1}{8} \zeta (3)}.

This problem has been solved by Julian Poon.

Tanishq Varshney - 5 years, 6 months ago

Log in to reply

Substitute tsin(x)t \to \sin(x). The integral becomes:

1201lntln(1t2)(1t)dt\frac { 1 }{ 2 } \int _{ 0 }^{ 1 }{ \ln { t } \ln { \left( 1-t^{ 2 } \right) } \left( \frac { 1 }{ t } \right) } dt

Substitute in the Taylor series for ln(1x)\ln (1-x):

1201lntn=1t2n1ndt=12n=11n01ln(t)t2n1dt-\frac { 1 }{ 2 } \int _{ 0 }^{ 1 }{ \ln { t } \sum _{ n=1 }^{ \infty } \frac { t^{ 2n-1 } }{ n } } dt=-\frac { 1 }{ 2 } \sum _{ n=1 }^{ \infty } \frac { 1 }{ n } \int _{ 0 }^{ 1 }{ \ln { (t) } t^{ 2n-1 } } dt

Finally, integrate by parts to obtain

12n=11n14n2=18n=11n3-\frac { 1 }{ 2 } \sum _{ n=1 }^{ \infty } \frac { 1 }{ n } \frac { -1 }{ 4{ n }^{ 2 } } =\frac { 1 }{ 8 } \sum _{ n=1 }^{ \infty } \frac { 1 }{ { n }^{ 3 } }

Julian Poon - 5 years, 6 months ago

Alternative approach could be (i know its too late :P) could be considering Beta function (Γ(a)Γ(b)Γ(a+b))(\dfrac{\Gamma(a) \Gamma(b)}{ \Gamma(a+b)}) in trignometric form and differentiating it with respect to a and b and then putting a = 0 and b = 1.

Harsh Shrivastava - 5 years, 5 months ago

Log in to reply

Great! That's the standard approach! Can you post the full solution?

Pi Han Goh - 5 years, 5 months ago

Log in to reply

@Pi Han Goh Γ(a)Γ(b)Γ(a+b)=20π/2sin2a1xcos2b1xdx\displaystyle \frac { \Gamma (a)\Gamma (b) }{ \Gamma (a+b) } = 2\int _{ 0 }^{ \pi /2 }{ \sin ^{ 2a-1 }{ x } \cos ^{ 2b-1 }{ x } dx }

Differentiating both sides first wrt a and then wrt b, we get

Γ(a)Γ(b)Γ(a+b)(((ψ(a)ψ(a+b))(ψ(b)ψ(a+b))ψ(a+b))=80π/2log(sin(x))log(cos(x))sin2a1xcos2b1xdx\displaystyle \frac { \Gamma (a)\Gamma (b) }{ \Gamma (a+b) } (((\psi (a)-\psi (a+b))(\psi (b)-\psi (a+b))-\psi '(a+b)) \displaystyle =8\int _{ 0 }^{ \pi /2 }{ log(sin(x))log(cos(x))\sin ^{ 2a-1 }{ x } \cos ^{ 2b-1 }{ x } dx }

Putting a = 0 and b = 1,

ψ(1)=80π2ln(sinx)ln(cosx)tanxdx-\psi ' (1) = 8\int^{\frac{\pi}{2}}_{0}\frac{\ln(\sin x) \ln(\cos x)}{\tan x} dx

Harsh Shrivastava - 5 years, 5 months ago

Log in to reply

@Harsh Shrivastava Even I got the same. Now your job is to prove it equal to 1/8 zeta(3).

Aditya Kumar - 5 years, 5 months ago

Log in to reply

@Aditya Kumar I don't have any idea how to proceed from here.

Harsh Shrivastava - 5 years, 5 months ago

Log in to reply

@Harsh Shrivastava Hint: What is the definition of a digamma function?

Pi Han Goh - 5 years, 5 months ago

Log in to reply

@Pi Han Goh @Harsh Shrivastava Want to join our discussion group where we talk bout integrals and series?

Pi Han Goh - 5 years, 5 months ago

Log in to reply

@Pi Han Goh Well I first need to clear my basics of series, so maybe after March or so, I'll join the group, BTW if you guys discuss from basics, then I'll surely join right now.

Harsh Shrivastava - 5 years, 5 months ago

Log in to reply

@Harsh Shrivastava Well, you can ask simple questions as well, we're happy to explain to you... Still want to join?

Pi Han Goh - 5 years, 5 months ago

Log in to reply

@Pi Han Goh Yeah sure! I'll join right after my school , I have to go to school now, Thanks!

Harsh Shrivastava - 5 years, 5 months ago

Log in to reply

@Harsh Shrivastava Go here

Pi Han Goh - 5 years, 5 months ago

Log in to reply

@Pi Han Goh When i enter my email address, it says already invited, what to do now? I am not able to access further.

Harsh Shrivastava - 5 years, 5 months ago

Log in to reply

@Harsh Shrivastava Check your email.

Pi Han Goh - 5 years, 5 months ago

@Harsh Shrivastava How's the registration coming along?

Pi Han Goh - 5 years, 5 months ago

Log in to reply

@Pi Han Goh I checked my mail but nothin:'s there regarding registration, please help!

Harsh Shrivastava - 5 years, 5 months ago

Log in to reply

@Harsh Shrivastava Step 1: GO here and type your email.
Step 2: GO check your email which you have typed in.
Step 3: Click that email link (for verification...). Done!

If none of these seem to work, register another email address and repeat steps 1 to 3.

Pi Han Goh - 5 years, 5 months ago

Problem 16:

Evaluate

01tarccos(t)1+t4dt.\large{\displaystyle \int^{1}_{0} \frac{t \arccos (t)}{ 1+t^4} dt}.

This problem has been solved by Pi Han Goh.

Tanishq Varshney - 5 years, 6 months ago

Log in to reply

This is very similar to Problem 5. Integrate by Parts: with dv=x1+x4dx,u=cos1(x)dv = \dfrac x{1+x^4} \, dx, u = \cos^{-1}(x) , then du=11x2dx,v=12tan1(x2)du =- \dfrac1{\sqrt{1-x^2}} dx, v = \dfrac12 \tan^{-1}(x^2) .

udv=uvvdu=cos1(x)12tan1(x2)x=0x=1  0+1201tan1(x2)1x2dx let x=sin(y)dy=dx1x2=120π/2tan1(sin2(y))dy let z=π2y=120π/2tan1(1cos2(z))dz refer to solution to problem 5=12π2tan1(12+1+12)=π4tan1(212)0.335426737379\begin{aligned} \int u \, dv &= &uv - \int v \, du \\ &=& \cancelto{0}{ \left . \cos^{-1}(x) \cdot \dfrac12 \tan^{-1} (x^2) \right |_{x=0}^{x=1}}+ \dfrac12 \int_0^1 \dfrac {\tan^{-1}(x^2)}{\sqrt{1-x^2}} \, dx \quad\quad && \text{ let } x = \sin(y) \Rightarrow dy = \dfrac{dx}{\sqrt{1-x^2}} \\ &=& \dfrac12 \int_0^{\pi /2} \tan^{-1} (\sin^2 (y)) \, dy \quad\quad && \text{ let } z = \dfrac \pi2 - y \\ &=& \dfrac12 \int_0^{\pi /2} \tan^{-1} (1 \cdot \cos^2 (z)) \, dz \quad\quad && \text{ refer to solution to problem 5} \\ &=& \dfrac12 \dfrac \pi2 \tan^{-1} \left ( \sqrt{\dfrac{\sqrt{1^2+1}+1}{2} }\right) \\ &=& \dfrac\pi4 \tan^{-1} \left( \sqrt{\dfrac{\sqrt2-1}{2}}\right) \approx 0.335426737379 \\ \end{aligned}

Pi Han Goh - 5 years, 6 months ago

Problem 17:

Prove that

(x2x4x3+x2x+1dx)÷(xx4x3+x2x+1dx)=3+52.\large \left(\int_{-\infty}^\infty \dfrac{x^2}{x^4 - x^3+x^2-x+1} \, dx \right) \div \left(\int_{-\infty}^\infty \dfrac{x}{x^4 - x^3+x^2-x+1} \, dx \right) = \dfrac{3+\sqrt5}2.

This problem has been solved by Surya Prakesh.

Pi Han Goh - 5 years, 6 months ago

Log in to reply

x2x4x3+x2x+1dx=x3+x2x5+1dx=0x3+x2x5+1dx+0x3+x2x5+1dx=0x2x31x5dx+0x3+x2x5+1dx=20x2x81x10dx\begin{aligned} \int_{-\infty}^{\infty}\dfrac{x^2}{x^4 - x^3 + x^2 - x+1} dx &=\int_{-\infty}^{\infty} \dfrac{x^3 + x^2}{x^5+1}dx \\ &= \int_{-\infty}^{0} \dfrac{x^3 + x^2}{x^5+1}dx + \int_{0}^{\infty} \dfrac{x^3 + x^2}{x^5+1} dx \\ &= \int_{0}^{\infty} \dfrac{x^2 - x^3}{1-x^5}dx + \int_{0}^{\infty} \dfrac{x^3 + x^2}{x^5+1} dx \\ &= 2 \int_{0}^{\infty} \dfrac{x^2 - x^8}{1-x^{10}} dx \end{aligned}

0x2x81x10dx=01x2x81x10dx+1x2x81x10dx=011+x2x6x81x10dx\begin{aligned} \int_{0}^{\infty} \dfrac{x^2 - x^8}{1-x^{10}} dx &= \int_{0}^{1} \dfrac{x^2 - x^8}{1-x^{10}} dx + \int_{1}^{\infty} \dfrac{x^2 - x^8}{1-x^{10}} dx \\ &= \int_{0}^{1} \dfrac{1+ x^2 - x^6 - x^8}{1-x^{10}}dx \end{aligned}

I just took the substitution t=1xt=\dfrac{1}{x} in the above steps.

So,

x2x4x3+x2x+1dx=2011+x2x6x81x10dx=1501t9/10+t7/10t3/10t1/101tdt=15(H1/10+H3/10H7/10H9/10)=15((ψ(9/10)ψ(1/10))+(ψ(7/10)ψ(3/10)))\begin{aligned} \int_{-\infty}^{\infty}\dfrac{x^2}{x^4 - x^3 + x^2 - x+1} dx &=2 \int_{0}^{1} \dfrac{1+ x^2 - x^6 - x^8}{1-x^{10}}dx \\ &= \dfrac{1}{5} \int_{0}^{1} \dfrac{t^{-9/10} + t^{-7/10} - t^{-3/10} - t^{-1/10}}{1-t} dt \\ &= \dfrac{1}{5} \left( H_{-1/10} + H_{-3/10} - H_{-7/10} - H_{-9/10} \right) \\&=\dfrac{1}{5} ((\psi(9/10) - \psi(1/10)) + (\psi(7/10) - \psi(3/10))) \end{aligned}

By euler reflection formula for Digamma function i.e. ψ(1x)ψ(x)=πcot(πx)\psi(1-x) - \psi(x) = \pi \cot(\pi x), we get

(ψ(9/10)ψ(1/10))+(ψ(7/10)ψ(3/10))=πcot(π/10)+πcot(3π/10)(\psi(9/10) - \psi(1/10)) + (\psi(7/10) - \psi(3/10)) = \pi \cot(\pi/10) + \pi \cot(3\pi/10)

So,

x2x4x3+x2x+1dx=15((ψ(9/10)ψ(1/10))+(ψ(7/10)ψ(3/10)))=π5cot(π/10)+π5cot(3π/10)\int_{-\infty}^{\infty}\dfrac{x^2}{x^4 - x^3 + x^2 - x+1} dx = \dfrac{1}{5} ((\psi(9/10) - \psi(1/10)) + (\psi(7/10) - \psi(3/10))) = \dfrac{\pi}{5}\cot(\pi/10) + \dfrac{\pi}{5} \cot(3\pi/10)

Similarly, evaluating the integral in the denominator, we get

xx4x3+x2x+1dx=2π5cot(3π/10) \int_{-\infty}^{\infty}\dfrac{x}{x^4 - x^3 + x^2 - x+1} dx = \dfrac{2\pi}{5} \cot (3\pi /10)

So finally

(x2x4x3+x2x+1dx)÷(xx4x3+x2x+1dx)=π5cot(π/10)+π5cot(3π/10)2π5cot(3π/10)=cot(π/10)+cot(3π/10)2cot(3π/10)=3+52\left(\int_{-\infty}^{\infty}\dfrac{x^2}{x^4 - x^3 + x^2 - x+1} dx \right) \div \left(\int_{-\infty}^{\infty}\dfrac{x}{x^4 - x^3 + x^2 - x+1} dx \right) \\ = \dfrac{\dfrac{\pi}{5}\cot(\pi/10) + \dfrac{\pi}{5} \cot(3\pi/10)}{\dfrac{2\pi}{5} \cot (3\pi /10)} = \dfrac{\cot(\pi/10)+\cot(3\pi/10)}{2 \cot (3\pi /10)}= \dfrac{3+\sqrt{5}}{2}

Note: cot(π/10)=5+25\cot (\pi/10) = \sqrt{5+2\sqrt{5}} and cot(3π/10)=525\cot (3 \pi/10) = \sqrt{5-2\sqrt{5}}.

Surya Prakash - 5 years, 6 months ago

Problem 18:

Evaluate

0log2(t)1+t2dt\large \int_{0}^{\infty} \dfrac{\log^2 (t)}{1+t^2} \, dt

This problem has been solved by Tanishq Varshney (first) and Sudeep Salgia (second) almost at the same time.

Surya Prakash - 5 years, 6 months ago

Log in to reply

Solution to problem 18\text{Solution to problem 18}

01ln2(t)1+t2dt+1ln2(t)1+t2dt\large{\displaystyle \int^{1}_{0} \frac{\ln^{2} (t)}{1+t^2} dt+\displaystyle \int^{\infty}_{1} \frac{\ln^{2} (t)}{1+t^2} dt}

In the latter integral t1tt\rightarrow \frac{1}{t}

The whole expression becomes

201ln2(t)1+t2dt\large{2 \displaystyle \int^{1}_{0} \frac{\ln^{2} (t)}{1+t^2} dt}

Now

01xadx=1a+101sxsxadx=01xalnsxdx=(1)ss!(a+1)s+1\large{\displaystyle \int _{ 0 }^{ 1 }{ { x }^{ a } } dx=\frac { 1 }{ a+1 } \\ \displaystyle \int _{ 0 }^{ 1 }{ \frac { { \partial }^{ s } }{ { \partial x }^{ s } } } { x }^{ a }\quad dx=\displaystyle \int _{ 0 }^{ 1 }{ { x }^{ a } } { \ln { ^{ s } } { x } }dx=\frac { { \left( -1 \right) }^{ s }s! }{ { \left( a+1 \right) }^{ s+1 } } }

we have

11+t2=r=0(1)rt2r\large{\frac{1}{1+t^2}=\displaystyle \sum^{\infty}_{r=0} (-1)^{r} t^{2r}}

201r=0ln2(t)(1)rt2rdt\large{\longrightarrow 2 \displaystyle \int^{1}_{0} \displaystyle \sum^{\infty}_{r=0} \ln^{2} (t) (-1)^{r} t^{2r} dt}

2r=0(1)r01t2rln2(t)dt\large{\longrightarrow 2 \displaystyle \sum^{\infty}_{r=0} (-1)^{r} \displaystyle \int^{1}_{0} t^{2r} \ln^{2} (t) dt}

4r=0(1)r(2r+1)3\large{4 \displaystyle \sum^{\infty}_{r=0} \frac{(-1)^{r}}{(2r+1)^{3}}}

Now for the answer

Use of Dirichlet Beta function

which gives 4β(3)=π38\large{4 \beta(3)=\frac{\pi^{3}}{8}}

Tanishq Varshney - 5 years, 6 months ago

Problem 21:

For b<1|b|<1, find the value of the integral below in terms of bb.

π/2π/2ln(1+bsinx)sinxdx \int_{-\pi /2}^{\pi /2} \dfrac{ \ln(1+ b \sin x)}{\sin x} \, dx

This problem has been solved by Surya Prakesh.

Gautam Sharma - 5 years, 6 months ago

Log in to reply

f(b)=π/2π/2ln(1+bsin(x))sin(x)dxf(b)=π/2π/211+bsin(x)dxf(b) = \int_{-\pi/2}^{\pi/2} \dfrac{\ln ( 1+ b \sin (x))}{\sin(x)} dx \\ f'(b) = \int_{-\pi/2}^{\pi/2}\dfrac{1}{1+ b\sin(x)} dx

Using Weierstrass Substitution i.e. t=tan(x/2)    sin(x)=2t1+t2t = \tan (x/2) \implies \sin(x) = \dfrac{2t}{1+t^2}.

f(b)=211dtt2+2bt+1=211dt(t+b)2+(1b2)2=21b2[arctan(t+b1b2)11=21b2[arctan(1+b1b2)arctan(1+b1b2)]=21b2[arctan(1+b1b2)+arctan(1b1b2)]=21b2[arctan(1+b1b)+arctan(1b1+b)]=π1b2\begin{aligned} f'(b) &= 2\int_{-1}^{1} \dfrac{dt}{t^2 + 2bt + 1}\\ &= 2\int_{-1}^{1} \dfrac{dt}{(t+b)^2 + (\sqrt{1-b^2})^2}\\ &= \dfrac{2}{\sqrt{1-b^2}} \Bigr[\arctan\left( \dfrac{t+b}{\sqrt{1-b^2}} \right) \Bigr|_{-1}^{1}\\ &=\dfrac{2}{\sqrt{1-b^2}} \Bigr[\arctan\left( \dfrac{1+b}{\sqrt{1-b^2}} \right) - \arctan\left( \dfrac{-1+b}{\sqrt{1-b^2}} \right)\Bigr] \\ &=\dfrac{2}{\sqrt{1-b^2}} \Bigr[\arctan\left( \dfrac{1+b}{\sqrt{1-b^2}} \right) + \arctan\left( \dfrac{1-b}{\sqrt{1-b^2}} \right)\Bigr] \\&= \dfrac{2}{\sqrt{1-b^2}} \Bigr[\arctan\left(\sqrt{\dfrac{1+b}{1-b}} \right) + \arctan \left(\sqrt{\dfrac{1-b}{1+b}} \right)\Bigr] \\&= \dfrac{\pi}{\sqrt{1-b^2}} \end{aligned}

I used the fact that arctan(x)+arctan(1/x)=π/2\arctan(x) + \arctan(1/x) = \pi/2.

So,

f(b)=π1b2f(b)=π1b2dbf(b)=πarcsin(b)+cf'(b) = \dfrac{\pi}{\sqrt{1-b^2}} \\ f(b) = \int \dfrac{\pi}{\sqrt{1-b^2}} db \\ f(b) = \pi \arcsin(b) +c

But f(b)=π/2π/2ln(1+bsin(x))sin(x)dx    f(0)=π/2π/2ln(1)sin(x)dx=0f(b) = \int_{-\pi/2}^{\pi/2} \dfrac{\ln ( 1+ b \sin (x))}{\sin(x)} dx \implies f(0) = \int_{-\pi/2}^{\pi/2} \dfrac{\ln ( 1)}{\sin(x)} dx =0

So, c=0c=0. Therefore, f(b)=πarcsin(b)f(b) = \pi \arcsin(b).

Surya Prakash - 5 years, 6 months ago

Problem 22:

Evaluate the integral

0πxsin(x)(cos2(x)+3)2dx.\large \int_{0}^{\pi} \dfrac{x \sin (x)}{(\cos^2 (x) + 3)^2} dx .

This problem has been solved by Aditya Kumar.

Surya Prakash - 5 years, 6 months ago

Log in to reply

I=0πxsinx(cos2x+3)2dxI=π20πsinx(cos2x+3)2dxcosx=tI=π2111(t2+3)2dtt=3tanxI=π336π6π6(cos2x+1)dxI=π318{34+π6}I=\int _{ 0 }^{ \pi }{ \frac { xsinx }{ { \left( { cos }^{ 2 }x+3 \right) }^{ 2 } } dx } \\ I=\frac { \pi }{ 2 } \int _{ 0 }^{ \pi }{ \frac { sinx }{ { \left( { cos }^{ 2 }x+3 \right) }^{ 2 } } dx } \\ cosx=t\quad \\ I=\frac { \pi }{ 2 } \int _{ -1 }^{ 1 }{ \frac { 1 }{ { \left( { t }^{ 2 }+3 \right) }^{ 2 } } dt } \\ t=\sqrt { 3 } tanx\\ I=\frac { \pi \sqrt { 3 } }{ 36 } \int _{ \frac { -\pi }{ 6 } }^{ \frac { \pi }{ 6 } }{ \left( cos2x+1 \right) dx } \\ \therefore I=\frac { \pi \sqrt { 3 } }{ 18 } \left\{ \frac { \sqrt { 3 } }{ 4 } +\frac { \pi }{ 6 } \right\}

In the first step, I used: abf(x)=abf(a+bx)\int _{ a }^{ b }{ f(x) } =\int _{ a }^{ b }{ f(a+b-x) }

Aditya Kumar - 5 years, 6 months ago

Problem 20:

Evaluate

0dx(1+x)3+1\large \int_{0}^{\infty} \dfrac{dx}{(1+x)^3 +1}

This problem has been solved by Gautam Sharma.

Surya Prakash - 5 years, 6 months ago

Log in to reply

Solution to problem 20

Let (1+x)=tI=dtt3+1(1+x)=t \\ I=\large \int \dfrac{dt}{t^3 +1}

By partial fractions

I=2t3(t2t+1)dt+13(t+1)dtI=12(t2t+1)dt2t16(t2t+1)dt+13(t+1)dtI=12((t12)2+34)dt2t16(t2t+1)dt+13(t+1)dtI=13tan12t1316ln(t2t+1)+13ln(t+1)+cI=13tan12x+13+16ln((x+2)2x2+x+1)+cI=\int \frac { 2-t }{ 3({ t }^{ 2 }-t+1) } dt+\frac { 1 }{ 3(t+1) } dt \\ I=\int \frac { 1 }{ 2({ t }^{ 2 }-t+1) } dt-\frac { 2t-1 }{ 6({ t }^{ 2 }-t+1) } dt+\frac { 1 }{ 3(t+1) } dt \\ I=\int \frac { 1 }{ 2({ (t-\frac { 1 }{ 2 } ) }^{ 2 }+\frac { 3 }{ 4 } ) } dt-\frac { 2t-1 }{ 6({ t }^{ 2 }-t+1) } dt+\frac { 1 }{ 3(t+1) } dt \\ I=\int \frac { 1 }{ \sqrt { 3 } } \tan ^{ -1 }{ \frac { 2t-1 }{ \sqrt { 3 } } } -\frac { 1 }{ 6 } \ln({ t }^{ 2 }-t+1)+\frac { 1 }{ 3 } \ln(t+1)+c \\ I=\int \frac { 1 }{ \sqrt { 3 } } \tan ^{ -1 }{ \frac { 2x+1 }{ \sqrt { 3 } } } +\frac { 1 }{ 6 } \ln\left(\frac { { (x+2) }^{ 2 } }{ { x }^{ 2 }+x+1 } \right)+c

Putting limits and evaluating we get π33ln23\frac{\pi}{3\sqrt3}-\frac{\ln2}{3}

Gautam Sharma - 5 years, 6 months ago

I used complex analysis

Nahom Assefa - 2 years, 7 months ago
×

Problem Loading...

Note Loading...

Set Loading...